wk 4 uworld

Réussis tes devoirs et examens dès maintenant avec Quizwiz!

WK4 MON--------->34 yo F presents w/ inquiries about progestin-IUD..pt says she's been using condoms..menses occur every 8 to 12wks w/ heavy vaginal bleeding and passage of clots along w/ intermittent spotting...pmh of trichomonad at 18...grandam had breast cancer..BMI is 44 - why is the progestin-IUD contraindicated in pt?

-CI dt menstrual bleeding pattern -unexplained abnormal vaginal bleeding could be from infections such as cervicitis/PID...it could be from endometrial polyp...it could be from endometrial hyperplasia or cancer w/ multiple risk factors for unopposed estrogen exposure such as obesity..chronic an ovulation....this abnormal bleeding causes for further evaluation before insertion -progestin IUD is CI in pts w/ active breast cancer...bc breast cancers are progesterone sensitive... -an unexplained vaginal bleeding requires further evaluation such as endometrial bx

WK4 MON--------->28yo F w/ sickle cell dz undergoes induction of labor and has spontaneou vaginal delivery...begin having vaginal bleeding and passing large fist sized clots...give oxytocin and pt begin having SOB..chest pain..bp 70/40...RR 28..hb6..platelts 80k -cause of acute decompensation?

-DIC Significant obstetric bleeding such as postpartum hemorrhage can cause DIC. This condition typically presents with bleeding and or thrombosis such as pulmonary embolus, thrombocytopenia and prolonged PT and PTT.

WK4 FRI--------->49 yo female present w/ unilateral worsening back pain since her hysterectomy..it non radiates..w/ cva tenderness..normal vitals and normal UA -cause?

-Hydronephrosis This patient post operative unilateral back pain, nausea with vomiting and costovertebral angle tenderness are most likely due to hydronephrosis from ureteral injury. With continued obstruction, patients have non radiating back pain and costovertebral angle tenderness. ureteral injury presents post operatively with urethral obstruction and subsequent hydronephrosis such as non radiating focal back pain unilateral costovertebral angle tenderness. Without prompt diagnosis and treatment irreversible renal damage can occur. The best test is an ultra sound Acute pyelonephritis may present with costovertebral angle tenderness and it's a complication of recent Foley catheter use. It typically have fever chills and abnormal urinalysis such as bacteria red and white blood cells. Nephrolithiasis can present with unilateral back pain nausea and vomiting patients typically have pain that radiates to the growing and microscopic haematuria on urinalysis renal vein thrombosis and subsequent unilateral renal infarction can occur in patients in a hypercoagulable state such as malignancy. In acute cases patients can develop back pain and costovertebral angle tenderness these patients typically have hematuria.

9yo boyr presents w/ gross hematuria..had a uri 2wks ago..uncle died from renal dz...he has htn... Likely finding on histo?

-Longitudinal splitting of the glomerular basement membrane Alport--- allports syndrome is a progressive disease that presents with recurrent he materia and boys under 10 years old often with a family history of renal failure and or hearing loss. Complement levels are normal and the renal biopsy shows longitudinal splitting of the glomerular basement membrane Good pasture syndrome is a progressive glomerular nefritis accompanied by pulmonary disease such as alveolar hemorrhage biopsy reveals linear IgG deposits in the GBM PSGN---biopsy and post strip glomerulonephritis reveals mesangial and glomerular capillary deposits of C3 and IgG patients typically have edema haematuria and hypertension. However symptoms usually develop a one to three weeks after streptococcal fair and gitis or impetigo, and C3 level is decreased. IGA nephropathy---is characterized by mesangial IGA deposition. Patients have concomitant growth material and upper respiratory symptoms such as rhinorrhea and sore throat.

WK4 THURS--------->59 yo M w/ ASCVD discontinued taking his statin bc of muscle pain... -next best step in lipid management?

-Offer moderate-intensity statin therapy - High intensity statin therapy can reduce the risk of myocardial infarction and ischemic stroke in patients with atherosclerotic cardiovascular disease. For patients who are intolerant of one high intensity statin, switching to a different high intensity or a moderate intensity statin should be considered.

WK4 TUE--------->32 yo M presents for routine preemployment physical...10yr ago dad died of sudden MI...bp left arm=180/105...bP in right arm=175/103...vmi 22..lungs clearBL..w/ non tender upper abd mass..Hb is 15.2 and creatinine 0.8 -appropriate next step?

-abd ultrasound - Most patients with autosomal dominant polycystic kidney disease are asymptomatic until age 30 to 40 when renal cysts become large and numerous. Bilateral upper abdominal masses are typically palpable on physical examination and hypertension is an early disease manifestation that typically precedes a progressive decline in renal function.

WK4 THURS--------->32 yo pF presents w/ dyspnea..dry cough...wheezing...nasal congestion.RR28...o2sat 95% -most appropriate med?

-albuterol and prednisone - The patient's cough wheezing and difficulty breathing are likely caused by an exacerbation of new onset asthma. X-ray findings may be normal or show lung hyperinflation due to chronic air trapping. New-onset asthma came present as an exacerbation with cough wheezing and shortness of breath. Treatments include albuterol/inhaled short acting beta agonist for bronchodilation and the systemic glucocorticoids such as prednisone to reduce airway inflammation.

WK4 THURS--------->36 yo M say for past 2 day..he wiped his butt and saw blood on the tp after pooping....he has normal bowel movements..no pmh...digital rectal exam showed traces of blood visible on gloves -nbsim?

-anoscopy -the pt has "minimal bright red blood per rectum{BRBPR)...described as small amt of bright red blood on toilet paper after pooping..most of benign dt hemorrhoids or ana fissures...but could also be dt serious issues such as proctitis...ulcers..colorectal polyps...cancer are possible...clues for a serious dz would include large amt of blood mixed w/ stool and systemic sxs such fever and weightless..changes on bowel habits or stool caliber..these sxs general indicted the need for a full visualization of the colon via a colonoscopy...also colonoscopy should be considered for pts 45 or older...for colorectal screening regardless of sxs -if the pt has hemorrhoids..start w/ conservative treatment such as stool softeners

WK4 MON--------->32 yo F presents w/ absent menses...since her vaginal deliver 4months ago that complicated w/ postpartum hemorrhage requiring transfusion and emergency SUCTION AND SHARP CURETTAGE..pt say she's been fatigue since returning to work and is bottle feeding...fsh and ash levels are normal -likely dx?

-asheman -dt the curettage...ASherman is the formation of intrauterine adhesion that can follow intrauterine surgery - Asherman syndrome the formation of intrauterine adhesions, often occurs following intrauterine surgery such as suction curettage. Patient typically have a Minorea, That does not respond to progesterone challenge due to obliteration of the endometrium.

WK4 FRI--------->40 yo F at 32 wks GA..concernced that baby isn't moving...non stress test shows baby's hr at 130/min w/ moderate variability and no decels..no acceleration after an hour despite vibroacoustic stimulation -nbsim?

-biophysiocal profile Parents would decreased fetal movement should undergo end to Non stress test fetal testing with the non stress test followed by a biophysical profile or a contraction stress test if the NST is non reactive. A biophysical profile should be performed if labor is contraindicated such as placenta previa or prior myomectomy.

WK4 THURS--------->81 yo M present w/ hypotensions....tachycardia..elevated cardia index(cardiac output)..w/ normal CWP an normal PCWP...SVR is dec -likely additional findings?

-bounding pulses This patient would likely pneumonia due to reduced breath sounds and crackles in the right lung has signs of septic shock including hypOrtension and tachycardia. Patient with septic shock developed hypOtension from peripheral vasodilation which leads to decreased systemic vascular resistance and capillary leak with loss of intravascular volume into the surrounding tissues which leads to borderline-low right atrial pressure and PCWP. To compensate for intravascular hypovolemia, the cardiac index and stroke volume increases leading to an increased pulse pressure which can manifest as bounding peripheral pulses. - A patient with distended neck veins likely has pooling in the Venus system which can occur in states of volume overload such as congestive heart failure. These patients were half elevated PCWP and right atrial pressures due to intravascular overload with a decreased cardiac index due to decrease contractility in the setting of cardiac muscle stretch from volume overload Pulses pardoxus...aw cardiac tamponade...pts have inc PVR and RAP and ventricular pressures w/ equalization of RAP,RVEDV and PCWP -s3=early chf and inc vol overload

WK4 FRI--------->A 33 year old woman at 30 weeks GA presents with fever nausea vomiting and right flank pain she is admitted to the hospital for acute pyelonephritis and started on Ivy ceftriaxone urine culture shows E coli that is sensitive to cephalexin ciprofloxacin fossil mice and nitrofurantoin and TMP as an X what is the most appropriate antibiotic?

-cephalexin Pregnant patients with acute pyelonephritis initially require Ivy broad spectrum antibiotics such as ceftriaxone. Those with symptomatic improvement such as afebrile for 48 hours can be transitioned to oral antibiotics, which are chosen based on fetal safety profile and urine culture sensitivity testing. Penicillin cephalosporins such as cephalexin and fosfomycin are safe during all trimesters.

WK4 TUE--------->22 yo F presents w/ recurrent syncopal episodes...the first time was a yr ago when Roomate committed suicide...and it happens when pt is emotional...bf syncope pt feels lightheaded...weak...and has blurred vision..it last about 3 minutes and ends w/ a rapid recovery to consciousness...vitals and ecg are normal... -nbsim?

-counterpressure maneuver General treatment measures in patients with vasovagal syncope include reassurance in education about the benign nature of the condition. Patient should be advised to avoid triggers and to use physical counterpressure maneuvers during the prodromal phase to avoid or delay an episode of syncope.

WK4 Tterm-47HURS--------->28 yo F presents w/ Brough to office by husband bc of strange behavior for past 6 months..refusing to eat any food thats not prepackaged dt fear of becoming ill..pt put locks on cabinets where she keeps her food and refuses to eat at restaurants..and examines food for possible contamination...continues to work but seldolmy interacts w/ coworkers..pts mood is anxious and tense -likely dx?

-delusional disorder - Delusional disorder involves one or more delusions in the absence of other prominent psychotic symptoms. Apart from the impact of delusional beliefs, the individual is still able to function. -w/ delusional disorders..theyre non bizarre delusions(false but plausible) -personlaity disorders are ego-syntonic..meaning that pts are not distressed by them.. -Personality disorders are characterized by lifelong patterns of behavior beginning in early adulthood they do not involve delusional beliefs. Paranoid personality disorder involves pervasive distrust suspiciousness of others such ass suspicious, mistrustful, secretive, isolated and questioning of the loyalty of family and friends.

WK4 THURS--------->37 yo F presents w/ ocasional episodes of nocturnal substernal chest pain that wake her during sleep...pain aw sweating...palpitations...and nausea,,pmh smoking cigarette..ecg shows st elevations in leads 1,avl,and v4-6 -best treatment?

-diltiazem Vasospastic angina results from hyperactivity smooth muscle, leading to intermittent coronary artery vasospasm. Calcium channel blockers such as diltiazem amlodipine cause coronary artery vasodilation and are the preferred pharmacologic treatment

WK4 FRI--------->34 yo F at 33wks FGA presents w/ signs of intraamniotic ifxn...fever..and fetal tachycardia of 170/bpm..baby also lying in transverse position -indications for delivery?

-fetal tachycardia Management of preterm premature rupture of membranes diagnosed at less than 34 weeks gestation is typically expected. However, in patients with overt signs of intra amniotic infection such as fever and fetal tachycardia, delivery is indicated to decrease maternal and neonatal morbidity.

WK4 THURS--------->16yo M present w/ eye injury after getting struck in eye by flying object while mowing lawn..has a foreign body sensation..pain...excessive tearing and dec vision eye -most suggestive of acute globe perforation?

-fixed teardrop pupil Open globe laceration is typically caused by small high velocity particles sent airborne by power tools, explosions, lawnmowers, or motor vehicle accidents. Large open globe lacerations may present with global deformity, extrusion of vitreous or iris or a visible entry wound. Other manifestations include a peaked or teardrop pupil asymmetric anterior chamber death loss of visual acuity or afferent pupillary response and reduce intraocular pressure. Hypopyon- A layering of inflammatory cells and their dependent portion of the anterior chamber. Perilimbal injection also called ciliary flush is characterized by dilation of the vasculature at the junction of sclera and cornea. These findings are seen in a variety of inflammatories such as in tear uveitis iritis and infection such as keratitis conditions. - Acute corneal opacification suggest angle closure glaucoma, which most commonly occurs in older individuals and is associated with headache nausea conjunctival redness and a fixed dilated pupil.

WK4 MON--------->33yo F g2p2 at 14wks gestation presents to prenatal visit..feeling well w/ no complaints...last baby was delivered at 30wks via c-section dt breech presentation and preeclampsia complications..vitals are normal -nbsim?

-give low dose aspirin -need asa bc she's high risk for preeclampsia since she experienced it in last pregnancy—asa inhibits platelet aggregation and helps prevent placental ischemia...since its believed that this dz results from abnormal vasoconstriction and inc platelet aggregation resulting placental infarction and ischemia

WK4 THURS--------->17 yo M presents w/ right thigh pain and swelling after being kicked my a soccer player in a game 4wks ago..now the R thigh is 2cm greater in circumference than the L..w/ a 3cm mobile mass anteriorly..pain reproduced on passive flexion of the knee -likely cause of the condition?

-heterotopic bone formation - The patient had an acute muscle injury, followed a few weeks later by the development of local pain and in indurated mass. This presentation is most consistent with myositis ossifications which is characterized by the formation of lamellar bone in extra skeletal tissues such as heterotopic ossification. Myositis ossificans is typically triggered by severe recent trauma such as muscle contusion orthopedic surgery. It can also be seen following significant neurologic events such as stroke or traumatic brain injury. And this pathophysiologic process increased expression of bone morphogenic proteins lead to in migration of spindle cells spindle stem cells mature into fibroblast chondrocytes osteocytes resulting in Cartlidge and metal metaplastic bone formation Myositis ossifications typically present as a painful firm mobile mask with local swelling it is most common in the quadriceps in brachialis. Alkaline phosphatase and inflammatory markers such as ESR are often elevated. X-ray may show calcification with radiolucent zones such as eggshell calcification but it's often non-diagnostic early course.

WK4 WED--------->70 yo F present w/ worsening constipationa..uses fingers to push into her vagina to poop. On valsalva maneuver , the posterior wall extends outside the hymen ring. -likely underlying cause?

-levator ani muscle complex injury Pelvic organ prolapse, the herniation of pelvic organs such as the rectum into the vagina occurs due to damage of the levator Ani muscle complex. Patient with prolapse of the posterior vaginal wall such as rectocele classically have chronic constipation and a vaginal bulge. -external anal sphincter issues--assoc w/ perineal laceration during vaginal delivery...assoc w/ bowel incontincence

WK4 MON--------->16 yo F present w/ opain during menses...pain in RLQ...radiates to the right flank...she has fatigue and nausea that start the days of moderate bleeding and resolves when cycle ends... -concerns for secondary(pathologic) cause of dysmenorrhea in the pt?

-location of pain - Dysmenorrhea or painful menses is common in reproductive age to women and is divided into primary also known as physiologic and secondary also known as pathologic causes. Clinical features suggestive of secondary cars such as endometriosis include women over 25 at onset unilateral pelvic pain, lack of systemic symptoms, and abnormal uterine bleeding.

WK4 MON--------->37 yo F presents for prenatal poointment...at 16wks GA...maternal serum quadruple test show elevated beta-hCG and elevated inhibit-A -cause of these lab results?

-meiotic nondisjunction of chr.21 -the maternal quadruple screen is performed in the second trimester(15-20wks)..women 35 and older at inc risk of fetal aneuploidy...inc bhcg..along w/ inc inhibin A suggests Down-syndrome...call-free fetal DNA can be offered if abnormal quadruple screening...an ultrasound should be done to evaluate for fetal anomalies.. -inc maternal serum alpha-fetal protein is assoc w/ neural tube defects and abd wall defects...folic acid def can lead to failure of primary neurulation..resulting in myelimeningocele...as for abd wall defects..gastroschisis is a paraumbilical bowel evisceration w/ no covering membrane....omphalocele is a peritoneum covering sac at the umbilicus

WK4 SAT--------->36 yo M is in MVA...bp os 150/90..bmi 24...glucose is 156..HbA1c is 5.5%... -nbim pt's glucose?

-no treatment Stress hyperglycemia is a transient elevation in blood glucose that occurs as a physiologic response to severe illness or injury. Mild elevations do not require treatment marketed elevations such as more than 200 are associated with increased mortality and should be corrected with short acting insulin, with the target glucose of 140 to 180.

WK4 SUN--------->Health committee decides A health committee decides to conduct a root cause analysis and construct a fishbone diagram is shown below. Which of the following best describes the primary purpose of this diagram?

-organize categories and causes of errors that undermine system performance -The Fishbone diagram cause-and-effect diagram is often used in root cause analysis to visualize organize the categories and causes of areas leading to a quality of care problem. Its main components include problem being analyzed, main categories influencing the problem, and the specific sub causes pertaining to each category.

WK4 TUE--------->65 yo M presenta w/ fatigue...poor appetite and Lower leg swelling...pmh of htn..t2dm..chronic cough...chest infix...lower extremity pad...pmh of 40pack yr smoking...PE show barely chest w/ wheeze...distended abd w/ liver palpated at 4cm below right costal margin..pitting edema and dilated tortuous veins over both lower extremities...manual pressure on the abd causes persistent distension of jugular veins..serum sodium 135 and creatinine 1.2 -likely cause pf pts edema

-pulmonary artery systolic pressure Cor pulmonale is impaired function of the right ventricle due to pulmonary hypertension they usually occurs due to chronic lung disease. Signs of right ventricular failure include elevated jugular venous pressure, right ventricular third heart sound, tricuspid regurgitation murmur calm, hepatomegaly with pulsatile liver, lower extremity edema, societies, and or plural effusion. Echocardiogram will show signs of increased right heart pressure and right heart catherization will show pulmonary systolic pressure greater than 25 mmHg.

WK4 TUE--------->29 yo F who had a vagnal delivery 3months ago presents w/ irregular vaginal bleeding..an enlarged uterus...chest pain and dyspnea for past 10 days...CXR show BL infiltrates of various shapes -most helpful in establishing dx?

-quantative B-hCg test -choriocarcinoma is a metastatic form of gestational trophoblastic neoplasia that may occur after a hydatidtiform mole...a normal pregnancy...or spontaneous abortion...the lungs most frequent site of metastatic...choriocarcinoma should be suspected in postpartem women w/ an enlarged uterus..irregular vaginal bleeding..pulmonary sxs...and multiple infiltrates on CXR...dx is confirmed w/ elevated BhCg level

WK4 FRI--------->35 yo male presents w./ pain that travels down his leg after lifting heavy object -nbsim?

-trial of NSAIDs - Lumbosacral radiculopathy is due to nerve root compression, most commonly by a herniated disc. The diagnosis is based primarily on clinical findings such as neural imaging such as MRI is not recommended in uncomplicated cases. Lumbosacral ridiculopathy resolves in most patients. Initial management include NSAId drugs and continued activity.

WK4 FRI--------->16 yo girl w/ pain in multiple joints..knee..hip..shoulder..wrist..hand joints....2days of fever...joint fluid shows 30k leukocytes -likelky cause?

-unprotected sex -nice sear neisseria gonorrhea is contracted via unprotected sexual intercourse. Although it usually causes symptomatic urogenital disease, a minority of patients develop disseminated gonococcal infection, which typically presents with either purulent monoarthritis or pustular rash, tenosovitis, and migrating arthralgia. Overlapping symptoms occasionally occur

WK4 THURS--------->49 yo F presents w/ uroine incointinence...urine loss when cough...laugh..bmi 31.. -cause of sxs?

-urethral hypermobility - Stress urinary incontinence occurs due to weak in pelvic floor muscles that can cause urethral hyper mobility and reduce bladder support. Patients typically have intermittent loss of urine from increased intra-abdominal pressure such as jogging or coughing. Bladder outlet obstruction or a neurogenic bladder due to diabetic neuropathy can cause overflow incontinence due to bladder overdistention from incomplete bladder emptying. Detrusor over activity causes inappropriate bladder spasms associated with urge incontinence patients have a sudden need to urinate followed by immediate involuntary loss of urine. Postmenopausal women have vulvovaginal atrophy and associated with urinary symptoms such as frequency urgency due to estrogen deficiency. Vesicovaginal fistula an abnormal communication between the bladder and vagina, is a complication following prolonged labor, pelvic surgery, or pelvic radiation therapy. These patients have constant leakage of urine because the bladder continuously drains through the vagina.

WK5 TUE---------> case control study

A case control study is designed by selecting both Patients with a particular disease or cases in patients without the disease control's and then determining their previous exposure status.

WK4 THURS--------->Postoperative atelectasis

A common postoperative complication (especially after chest or abdominal surgery), which often occurs within 72 hours of surgery. Symptoms depend on the extent of atelectasis. Clinical findings include a dull percussion note, diminished breathing sounds, and decreased fremitus over the affected lung.

WK4 FRI--------->breast approach

A complex appearing breast mass has features such as thick walled, septated, solid and cystic components are suspicious for cancer and automatically require core needle biopsy. In contrast simple breast cyst appear thin walled anechoic such as fluid filled and without echogenic debris or solid components if the fluid is bloody or the mass persist after a fine needle aspiration is performed the patient may be at increased risk for breast cancer and should undergo a core needle biopsy which can provide a histologic diagnosis if aspiration yields none bloody fluid after a fine needle aspiration such as clear green or Gray and then mass resolves completely, patients are at low risk for cancer and require no additional management.

WK4 FRI--------->sheehan

A condition of hypopituitarism caused by ischemia and necrosis of the pituitary gland due to severe postpartum hemorrhage

acute bronchitis

A cute bronchitis is a common cause of cough that may be productive of appealant blood tin sputum. Upper respiratory viral infection is the tip is typically the ideology and symptoms are usually self-limited. Symptomatic treatment and close clinical follow-up are the best management strategies.

WK5 WED---------> Anorexia nervosa

A dietary history to assess for anorexia nervosa should be obtained in any patient with a stress fracture, low body weight, and distress at having to limit physical activity. Patient with anorexia nervosa are at risk for stress fractures due to decreased bone mineral density. Clinical features of anorexia nervosa includes a BMI less than 18.5 in fear of weight gain and distorted body image. The initial management of small tibial stress fractures include rest, analgesia, and stabilization with a splint or brace if necessary. Surgical intervention could be necessary for severe fractures or for lack of healing or severe pain despite conservative treatment.

WK4 SAT--------->cyproheptadine

A first-generation H1 antihistamine with nonselective antiserotonergic activity. It also has anticholinergic and antidopaminergic activity at higher concentrations. Common indications include carcinoid syndrome, serotonin syndrome, and cold-induced urticaria.

WK5 Thursday---------> Auto immune encephalitis

A group of inflammatory CNS disorders caused by immune mediated destruction or damage to intracellular or extracellular neuron or components such as anti-NMDA receptor and cephalitis. Clinical features can vary widely depending on the ideology but generally resemble those of infectious encephalitis such as neuropsychiatric manifestations, dyskinesia, dystonia, seizures, and other focal neurological symptoms. Anti-NMDA receptor is cephalitis is an autoimmune encephalitis syndrome characterized by psychiatric symptoms such as anxiety psychosis cognitive dysfunction such as memory impairment, seizure, autonomic instability, dystonia, and rigidity. It occurs most commonly in young women median age of 21 and is associated with ovarian teratoma and more than 50% of cases. Aseptic meningitis typically presents with a headache, fever, photophobia. Meningitis unlike encephalitis classically presents with normal cognitive function, unlike the altered mental status seen in this patient.

WK5 SAT--------->mature cystic teratoma v tuboovarian abscess

A mature cystic teratoma may present with abdominal pain due to mass effect or acute a torsion But does not cause fever and leukocytosis. Ultrasound typically reveals an ovarian cyst with solid components and hyperechoic nodules, and calcifications. Add to bowl ovarian abscess is a complication of pelvic inflammatory disease, a poly microbial infection of the upper genital tract. Patients typically have fever, diffuse abdominal pain that gradually worsens on one side in both the uterine and adnexal tenderness. Ultrasound typically shows a complex multiloculated adnexal mass with thick walls and internal debris.

WK4 SAT--------->dantrolene

A postsynaptic skeletal muscle relaxant that acts by inhibiting the ryanodine receptor on the sarcoplasmic reticulum and thereby prevents the release of calcium into the myocyte cytoplasm. The result is impaired muscle contraction. Used to treat malignant hyperthermia, neuroleptic malignant syndrome, or severe spasticity. dan directly deals w/ distressed muscles

WK4 SUN---------> precision and accuracy

A precise tool is one that consistently provides a very similar or identical value in measuring a fixed quantity. An accurate tool is one that provides a measurement similar to the actual value or within a specified range of reference value as reflected by a gold standard measurement.

WK5 SAT---------> Abnormal uterine bleeding

A progesterone withdrawal test is used to evaluate for secondary amenorrhea such as no menses for more than six months and patients were previously irregular menses to determine if the amenorrhea is from the estrogen levels such as no bleeding after progesterone. This patient has continue bleeding while on oral contraceptives suggesting high estrogen levels therefore this test is not indicated. Women under 45 with abnormal uterine bleeding who have failed medical management such as oral contraceptives require evaluation for endometrial hyperplasia/cancer with an endometrial biopsy.

WK4 FRI--------->cushing reflex suggests brain stem compression

htn...bradycardia...resp. depression

WK4 TUE--------->gold standard screening chlamyidia and gonorro

if chlam and gon results are pending...tz empirically w/ doxy and ceftriaxone

WK4 FRI--------->A man was rushed to the emergency room after an injury his fiance told the doctors do not provide a blood transfusion because he is a Jehovah's witness. Next best step in management?

in the absence of an advanced directive, a lifesaving blood transfusion can be given to a Jehovah's witness who lacks decision making capacity Most jehovah witnesses carry advanced directive cards that explicitly documents the individuals refusal of blood transfusion

WK5 MON---------> 22-year-old man presents to the office with intermittent rainy pain that has been increasing for the past eight weeks since a recreational soccer tournament where he had an injury causing a sensation of catching in the knee when walking. X-ray of the knee is normal which of the following is the best nap next step and management of this patient? MRI of the knee

acute knee pain associated with catching or reduced range of motion suggest a meniscus tear. Persistent symptoms in patients with suspected meniscal injury should be evaluated by MRI, and surgical consultation is advised for significant tears.

WK5 SUN--------->

all women should be assessed for depression at their postpartum follow up visits. Treatment options for postpartum depression include psychotherapy and or pharmacotherapy. Selective serotonin reuptake inhibitors are used as first line therapy.

WK4 SAT--------->baby born at 32wks has normocytic-normochromic rbc

anemia of prematurity dt impaired EPO production

WK4 FRI--------->duodenal atresia

assoc w/ trisomy 21 and VACTERL Do I know atresia a congenital anomaly causing complete bowel obstruction, present on fetal ultrasound as a fluid filled stomach and duodenum... double bubble.. with associ polyhydraminos. Because it is commonly seen in Down syndrome and VACTERL= vertebral anal atresia ...cardiac ...tracheoesophageal fistula ...esophogeal atresia Renal ...limb association, the presence of duodenal atresia requires evaluation of other fetal malformations such as ventricular septal defects.

Acute HIV infection

Acute HIV infection often causes a mononucleosis like syndrome consisting of fever, night sweats, lymphadenopathy, arthralgias, and diarrhea. Important diagnostic clues include oral observations and a transient rash such as Oliver red macular lesions. Provider should have no threshold for HIV testing even when traditional HIV risk factors are absent.

Acute a calculus cholecystitis

Acute a calculus cholecystitis is an acute inflammation of the gallbladder in the absence of gall stones that is most commonly seen in hospitalized and critically ill patients.

WK5 SAT---------> The treatment for unstable angina and nine ST segment elevation myocardial infarction is the same because both can cause acute coronary syndrome.

Acute coronary syndrome is due to unstable angina or non-ST elevation myocardial infarction is managed with antiplatelet and anti-thrombotic agent, beta blockers, nitrates, and high intensity statins. Fibrinolytic therapy is not used in patients with non-ST elevation acute coronary syndrome.

WK5 TUE---------> Acute fatty liver of pregnancy

Acute fatty liver of pregnancy presents in the third trimester with herpetic inflammation such as epigastric right upper quadrant pain leukocytosis, elevated aminotransferase's and fulminant liver failure such as profound hypoglycemia. Due to high maternal and fetal mortality rates, management is immediate delivery. Emergency cholecystectomy is performed for complicated acute cholecystitis such as gallbladder perforation or gangrene, which may present with postprandial right upper quadrant pain and an elevated alkaline phosphatase level.

WK5 SUN---------> Acute fatty liver of pregnancy

Acute fatty liver pregnancy typically occurs in the third trimester due to micro vesicular fatty infiltration of a parasite leading to liver inflammation such as right upper quadrant pain in elevated aminotransferase is in fulminant liver failure such as profiled hypoglycemia and thrombocytopenia. Management is immediate delivery.

WK4 SUN---------> necrotizing mediastinitis

Acute necrotizing mediastinitis is a life-threatening complication of untreated retropharyngeal abscess that presents with fever, chest pain, and dyspnea. Requires urgent surgical drainage to prevent spread to the posterior mediastinum which may cause lethal plural and pericardial effusion.

WK5 SUN---------> Nitrofurantoin associated with drug induced lung injury

Acute nitrofurantoin induced pulmonary injury is due to hypersensitivity that can present with fevers shortness of breath, dry cough, erythematous rash, period symptoms usually begin 3 to 9 days from the medication initiation. Bilateral Bibasilar opacities and pleural effusion's are common. Treatment involves cessation of nitrofurantoin.

WK5 SAT---------> Practice guidelines for burn care and management of genital burns

After initial stabilization burn patient who require aggressive fluid resuscitation such as due to burns covering a large total body surface area should undergo urethral catheterization as soon as possible before those areas of the body become swollen and making it difficult to catheterized a patient Following your retro catroization, treatment of the patients burned involves copious irrigation and gentle goals debridment of affected areas. This is followed by application of topical and tiemicrobial agent such as topical antibiotics and nonstick dressings.

WK4 SUN--------->A 14-year-old boy is brought to the emergency department after a head injury an hour ago while playing hockey he hit his head on the ice immediately developed a headache followed by a single episode of vomiting he remained awake and has symptoms resolved he is unable to cite the months backwards he has a game tomorrow what is the best recommendation?

After two days of rest, followed by gradual progression of activity intensity Concussion or mild traumatic brain injury is characterized by neurologic symptoms such as headache difficulty concentrating without structural intracranial injury. Initial treatment is with cognitive and physical rest for 24 to 48 hours, followed by a gradual return to play with slowly increasing physical intensity.

WK4 SUN--------->A 38-year-old man comes today emergency department we're just swelling and pain his tongue appears elevated and displaced posteriorly red and swollen after biting on and unpopped popcorn kernel which of the following is the immediate complication?

Airway obstruction Ludwig angina is a rapid progressively progressive cellulitis of the submandibular and sublingual spaces. Airway obstruction can occur due to displacement of the tongue posteriorly.

Lyme arthritis

Amano articular effusion of the knee in an adolescent is most often caused by trauma, bacterial infection, or inherited blood disorder. Less common causes include late Lyme disease, juvenile arthritis, and serum sickness, all of which are typically accompanied by a preceding ongoing rash.

Hepatitis B markers

Among healthy adults, acute hepatitis B virus infection SF limited in more than 95% of cases. In the recovery phase, serology typically demonstrates hepatitis B surface antibody which promotes immunity, IgG hepatitis B core antibody and hepatitis B E antigen antibody

als

Amyotrophic lateral sclerosis often causes weakness of the muscles of respiratory resulting in a restrictive pattern hospital amatory such as reduced final capacity, forced expiratory volume in one second FEV1, FVC within normal or increased FEV1 to FVC ratio, with reduce maximal inspiratory and expiratory pressure. The diffusion capacity for carbon monoxide, a measurement of alveolar gas exchange is preserved.

WK4 SAT--------->benztropine

An anticholinergic agent that competitively antagonizes muscarinic acetylcholine receptors. About 1/2 as potent as atropine. Also has some antihistamine effects. Commonly used to treat extrapyramidal symptoms associated with antipsychotic drugs and as an adjunctive treatment for Parkinson disease.

WK4 SAT--------->uterine inversion

An obstetric emergency that can follow vaginal delivery, in which the uterine fundus collapses into the endometrial cavity, which turns the uterus partially or completely inside out. Risk factors include nulliparity, cord traction, fundal pressure during the third stage of labor, and a retained placenta. Manifests with severe postpartum hemorrhage and a round, protruding mass from the cervix or vagina.

WK4 SUN---------> Mean, median, mode,

An outlier is defined as an extreme and unusual value observed in a dataset... The main is very sensitive to outliers and easily shift towards them. The median and the mode are more resistant to outliers.

WK5 SUN--------->anorectal fistulas

Anal rectal fistula's are most often due to rupture of. Anal abscess with formation of a persistent sinus tract. Symptoms include pain with defecation and chronic discharge. Management requires surgical intervention. Some perianal fistula's associated with Crohn's disease can be managed with antibiotics and medical treatment of the underlying inflammatory bowel disease. For fistula's arising from a perianal abscess, non-surgical treatment has a high failure rate.

WK5 TUE---------> Cohort studies

And cohort studies a group of exposed and an expose subject is followed overtime for development of the outcome of interest.

WK5 TUE---------> Cross-sectional study

And cross-sectional study, risk factor and I'll come are measured simultaneously and a particular point in time I Snapchat study. And other study designs a certain period separates the exposure from the outcome.

WK4 THURS--------->Gi bleeds

Angiodysplasia(lower GI bleed) Gastrointestinal bleeding → Etiology A common degenerative disorder of the gastrointestinal vessels, often causing gastrointestinal bleeding in the stomach, duodenum, jejunum and colon. Mostly consists of venous malformations, although arteriovenous and arterial changes may also occur. Associated with aortic stenosis. Dieulafoy lesion(upper GI bleed) Gastrointestinal bleeding → Etiology A condition in which mucosal trauma can lead to major GI bleeding due to an abnormal artery in the submucosa of the proximal stomach wall.

WK4 SUN---------> 55-year-old businessman presents with worsening joint pains for the past several months complaining of fingers and hands feeling tight when wakening in the morning 10 pound weight loss in the past six months metacarpophalangeal and proximal inter-phalangeal joints are both hands show mild boggy swelling with tenderness on gentle squeezing. Lab show elevated ESR what is the most appropriate step in evaluating the patient?

Anti-cyclic sertralineaided peptide antibody essay Anti-CCP Rheumatoid arthritis causes chronic symmetric polyarthritis predominantly affecting the small joints of the hands. Rheumatoid factor law specifically and cyclic sealed truly need a peptide antibodies hi specifically are the primary serologic markers for our a. Patients who have negative essays for both markers may have a more favorable prognosis. Spondyloarthropathy such as ankylosing spondylitis and reactive arthritis are both associated with HLAB 27 and are characterized by asymmetric oligo arthritis involving the large joints such as Denise and the situs such as inflammation of the insertion of ligaments and tendons and back pain. Hand involvement is rare. Reactive arthritis typically occurs acutely following a G.I. or genitourinary infection such as chlamydia and resolves within a few months. Parville B 19 causes symmetric arthritis that affects the hands feet knees it may resemble RA but the symptoms usually results spontaneously within a few weeks. Gout is caused by deposition of uric acid crystals in the synovial space the incidence correlates roughly with the serum uric acid levels. However gout most commonly presents for episodic a cute mono arthritis what dramatic redness and warm involving the toes ankles are knees.

WK4 SUN---------> 28-year-old woman G2 P0 comes to the office following a miscarriage her mother has hypothyroidism and is treated with levothyroxine. Her TSH level is 7.2 and free T4 is normal. Which of the following autoantibodies most likely associated with this patient's medical history?

Anti-thyroid peroxidase antibody Anti-TPO antibodies are present in more than 90% of patients with Hashimoto thyroiditis, high titers of anti-TPOR associated with increased risk of progression to overt hypothyroidism. High titers of anti-TPO are also associated with increased risk of miscarriage in both euthyroid and hypothyroid women. The risk of recurrent pregnancy loss is also increase in patients with antiphospholipid antibody syndrome. Anti-cardiolipin antibodies and lupus anticoagulant are typically checked in women with recurrent pregnancy loss. APS is not associated with thyroid dysfunction or goiter.

WK5 SUN--------->GCA

Aortic aneurysms can occur in patients with giant cell temporal arteritis due to the involvement of the branches of the aorta

WK5 WED---------> How can you reduce hospital remission for hospitals high-risk patients?

Arranging telephone-based follow up of patient shortly after discharge Hospital re-admissions are used as a quality metric because they are often preventable through improve patient communication and follow up. Telephone-based outreach shortly after discharge reduces readmission rates by preventing loss to follow up, increasing patient engagement proactively identifying and addressing potentially serious concerns and care coordination which effective at transitioning between inpatient outpatient care

WK5 SAT--------->LUNG INFXNS

Aspergillus primarily affects immunocompromise patients in can caused a classic triad a fever pleuritic chest pain in her mouth disease. Multilobar nodular or cavitary infiltrates may occur. The previously healthy patient with recent influenza more likely has community acquired methicillin resistant staph aureus. Histoplasma pulmonary infections usually present with might with mid subacute symptoms including fever myologist cough and chest pain. Focal pulmonary infiltrates are common and may be mistaken for bacterial community acquired pneumonia. This patient with acute severe symptoms after influenza more likely has community acquired methicillin resistant staph aureus Secondary bacterial pneumonia is the most common influenza complication and should be suspected when fever and pulmonary symptoms worsen after initial improvement. Most cases occur in patients over 65 but community acquired methicillin resistant staph aureus has a predilection for a young patient with the recent influenza. This pathogen causes rapidly progressive necrotizing pneumonia with high fever productive cough often with hemoptysis, leukopenia and multilobar cavity infiltrates.

WK4 TUE--------->26 yo F at 20wks GA presents w/ non-productive cough..SOB for the past month..its worsened at night...has a pmh of childhood asthma...denies being around smokind -likely dx?

Asthma Presentation of asthma similar impregnate and non-pregnant patient includes intermittent cough wheezing shortness of breath and chest tightness that all worsen with physical activity adequate control such as inhaled steroids is recommended to prevent maternal and fetal complications.

Afib

Atrial fibrillation is recognized on ECG buy an absence of organize P waves and irregularly irregular are are intervals. It is most commonly caused by a topic electrical foci within the pulmonary veins

WK5 SAT---------> Atrophic vaginitis

Atrophic vaginitis is a genitourinary syndrome of menopause it is due to low estrogen levels which cause decrease vulvovaginal tissue elasticity in blood flow. Patient may have resulting dyspareunia due to narrowing of the vagina introitus and dryness from loss of natural lubrication. Treatment is white lubricants or moisturizers vaginal estrogen is used for persistent or severe symptoms.

WK4 SUN---------> celiac disease is an autoimmune disorder characterized by intestinal hypersensitivity to gluten, a protein found in grains wheat rye and barley. Gluten triggers an auto immune reaction and the production of auto antibodies that target tissue transglutaminase within the small intestines. Manifestations include symptoms of malabsorption such as vitamin deficiencies, anemia, fatigue, weight loss and diarrhea.

Auto immune process Celiac disease can present with a diverse array of symptoms related to malabsorption such as diarrhea, microcytic anemia, vitamin D deficiency or autoimmune inflammation such as peripheral neuropathy atrophic glossitis. Peripheral neuropathy can occur in patients with HIV either due to neurotoxic affects of antiretroviral medication or due to mitochondrial dysfunction related to virus activity. Cipro and painful mucocutaneous ulcers, rather than atrophic glossitis can be seen patient will also have normocytic anemia in addition most patients with HIV have constitutional symptoms and lymphadenopathy.

WK4 FRI--------->24 yo F underwent emergency section delivery...2 days later she has induration and redness at the incision site along w/ fever..Foley catheter draining blood -likely cause?

Bacteria contamination of the incision superficial surgical site infix such as cellulitis...can occur after cessation and present w/ past part fever 38C or higher more than 24hrs after delivery as wells incisional induration and eyrthema..RFs include obesity..emergency c section such as inadequate skin antisepsis or abx prophylaxis.. Septic pelvis thrombophelphlebitis can cause postpartum fever due to injury, thrombus, and hematogenous spread of infection to the pelvic veins such as ovarian veins patients are typically initially treated for suspected endometritis but have relapsing-remitting fevers and persistent abdominal pain despite antibiotics. patients with mastitis typically have high fevers malaise and unilateral breast redness it is caused by obstruction inflammation and infection of breast ducts Patient to undergo C-section delivery are at risk fundsdometritis, a polymicrobial infection of the entry uterine cavity in common cause of postpartum fever. Patients with endometritis typically have uterine fundus tenderness periods.

WK5 SUN---------> Pediatric bacterial meningitis

Bacterial meningitis in infants came present now specifically with fever lethargic inconsolable nurse and poor feeding. Physical examination may reveal about a bulging fontanelle. And hemodynamically stable infants, lumbar puncture should be performed prior to antibiotic therapy to identify the pathogen and guide treatment.

WK4 FRI--------->Bartholin gland carcinoma

Bartholin gland cyst and abscess → Differential diagnoses A rare type of vulvar carcinoma that arises from a Bartholin gland. Primarily occurs among postmenopausal women and typically presents with gradual, painless enlargement of the gland. Diagnosis is made via histologic examination of a biopsy specimen. Treatment involves surgical resection.

Tues wk 6 bppv

Benign paroxysmal positional vertigo causes brief episodes of vertigo triggered by changes in the head position such as looking up or bending down. Triggering an episode by using provocative hair maneuver such as Dix Hallpike is diagnostic.

WK4 FRI--------->biliary atresia..think babies!!!

Biliary atresia Congenital visceral malformations → Biliary tract malformations A rare neonatal condition characterized by progressive fibrosis and obliteration of the extrahepatic biliary tree that leads to cholestasis, jaundice, acholic stools, and dark urine. This condition often progresses to liver failure requiring transplantation.

WK4 FRI--------->Choledochal cyst=biliary cyst

Biliary cancer→ Differential diagnoses Biliary cyst A condition characterized by cystic dilation of the biliary tree. Manifestations in infancy include obstructive jaundice and a palpable mass in the right upper quadrant. Older children and adults can develop intermittent biliary obstruction and recurrent pancreatitis. Associated with an increased risk of cholangiocarcinoma. Biliary atresia presents in early infancy with obstructive jaundice such as alcohol stools dark urine due to fibro obliteration of the extrahepatic biliary tree including the gallbladder. Carcinoma of the pancreatic head can cause obstructive jaundice and the nontender palpable mass related to an enlarged gallbladder such as Courvoisier sign Focal nodular hyperplasia is a solid liver lesion usually seen in women it is typically asymptomatic but may cause right upper quadrant pain pancreatic pseudo cyst which may cause abdominal pain vomiting and sometimes elevated bilirubin and light pace levels develop slowly over several months in patients with a history of either acute or chronic pancreatitis biliary cyst or congenital dilations of the biliary tree can initially present with complications such as Cholestasis and pancreatitis due to distal biliary obstruction leading to a reflux of bowel and pancreatic fluid

WK4 THURS--------->biliary colic

Biliary colic Cholelithiasis → Clinical features A condition characterized by dull pain in the right upper quadrant of the abdomen that often radiates to the right shoulder or scapula. Can be associated with nausea and vomiting. Caused by impaction of a gallstone in the neck of the gallbladder. Often triggered after a meal and typically lasts < 6 hours with spontaneous resolution.

WK5 MON---------> A three year old boy what did George syndrome is BART to the emergency department after five minutes of generalized tonic clonic seizures he had morning he has severe morning headaches and his family immigrated from another country to obtain and repair tetralogy of Fallot His temperature is 101 what is the most likely cause of this patient symptoms?

Brain abscess Brain abscess in children frequently presents with headache fever focal neurologic deficit and seizure. Cyanotic congenital heart disease is a risk factor for brain abscess due to hematogenous spread of the bacteria. Megalo blastoma is a pediatric tumor most commonly found in the cerebellum that can present with nocturnal or morning headaches and vomiting cerebella dysfunctions manifest ass in coordination and or ataxia

WK5 MON---------> 44-year-old man is brought to the ER prescribed Haldol and valproate over the next 10 days of admission he is found lying and confused and sweating and stiff temperature is 100.2.6 which of the following medications would be appropriate to administer?

Bromocriptine Neuroleptic malignant syndrome is a rare but potentially life-threatening emergency associated with the use of antipsychotics/neuroleptics. A patient whose condition does not improve with cessation of the antipsychotic and intensive supportive care can be treated with benzodiazepines, dopamine agonist, and or dantrolene.

WK4 FRI--------->

CARBOPROSt-trimethamine..also helps w/ utrine atony but its contraindicated in pts w/ asthma bc it casue bronchospasm

WK5 MON---------> A 26-year-old man presents to the ER due to six weeks of intermittent lower abdominal pain and cramps accompanied by rectal urgency bloody diarrhea nausea and decreased appetite. Temperature is 101.3 blood pressure is 90/50 bowel sounds decreased abdomen is distended rebound tenderness and muscle rigidity absent leukocyte count 31,000 next best step in management?

CT scan of the abdomen Diagnosis of toxic megacolon requires a radiographic evidence such as abdominal CT of colonic dilation greater than 6 cm along with manifestations of systemic toxicity such as fever leukocytosis and hemodynamic instability

WK4 SUN---------> chlamydia

Can cause dysuria in sterile pyuria such as white blood cells present but no bacteria do to urethritis, but patients typically have concomitant acute cervicitis such as cervical friability mucopurulent discharge. Lymphogranuloma beryllium is caused by chlamydia truckle Mattis L1 through L3 and presents with small painless ulcers follow by paying for suppurative inguinal lymphadenopathy such as buboes.

Cerebral amyloid angiopathy related hemorrhages

Cerebral amyloid angiopathy is associated with Alzheimer's dementia and occurs due to the beta amyloid deposition in the walls of small to medium cerebral arteries that lead to increase virginity of the vessels. The most common manifestation is a spontaneous lobar intracranial hemorrhage.

WK4 TUE--------->55yo M present to the ER dt sudden onset of palpitations and chest tightness..pmh of htn..dm...gout...cardiac monitor show fib..rate of 120-140....bp=112/70//o2sat of 92%...pt unresponsive...no palpable pulse over the carotid or femoral arteries..cardiac monitor shows fib at the same rate...nbsim?

Chest compressions - Pulseless electrical activity or asystole should be managed with an uninterrupted(CPR) cardio pulmonary resuscitation along with vasopressors to maintain adequate cerebral and coronary perfusion. Potential reversible causes of PEA should be investigated. Defibrillation or synchronized cardioversion has no role in the management of these patients. Arterial blood gas analysis should be performed while CPR is being administered to assess for and correct hypoxia as a potential reversible cause of PEA. IV lidocaine is a line ani-arrhythmic agent used for HDS monomorphic VT. IV epinephrine is used to TX PEA -defib delivers energy randomly during the cardiac cyclew/out synchronization to QRS complex...used in VFIB and pulseless VTACH -SCV delivers energy synchronized to the QRD complex. It is used in symptomatic or sustained monomorphic VTACH(unresponsive to antiarrhythmics) and HDUS fib w/ rapid ventricular response.

WK4 SUN---------> Pharyngitis approach

Children with pharyngitis and no viral signs such as rhinorrhea cough should undergo group a streptococcus rapid antigen detecting testing. Due to the risk of a cute rheumatic fever with entropy with untreated group a strep, a throat culture is performed to confirm a negative result.

WK5 SAT---------> management of Community acquired pneumonia in infants and children older than three months of age

Children with pneumonia Unappropriate antimicrobial therapy who have no improvement after 48 to 72 hours require evaluation with chest x-ray to assess for complications such as parapneumonic a fusion. Drainage is indicated for moderate or large effusion's or those causing respiratory distress.

WK5 MON---------> A 75-year-old man is brought to the ER due to worsening generalized abdominal pain and nausea for the past two days physical exam shows abdominal distention and a fused tenderness without rigidity or rebound tenderness x-ray shows a coffee bean shaped large intestine what is the most likely risk factor for this condition?

Chronic constipation Sigmoid volvulus occurs when a segment of the sigmoid colon twist on its mesentery, forming a close loop obstruction that often appears on the abdominal x-ray as dilated, Inverti, U-shaped loop/coffee being signed. Chronic constipation and colonic dysmotility are risk factors. Sigmoid volvulus occurs when a segment of the sigmoid colon twist on its mesentery forming a close the obstruction with continue gas formation by the intraluminal bacteria, the lumen of the obstructed bowel loop gradually expands this leads to slowly progression over two days abdominal distention frequent discomfort accompanied by loss of appetite nausea vomiting or obstipation.

WK5 WED--------->cgd

Chronic granulomatous disease is due to a defect in NADPH oxidase, resulting in decreased formation of super oxide ANIONS and inability of phagocytes to kill certain bacterial and fungus pathogens. The presentation include recurrent abscesses and pulmonary infections caused by catalase positive organisms such as Boko Darya pseudomonas and granuloma formation. Decrease immunoglobulin production as seen in X link agammaglobulinemia generally presents with recurrent Sino pulmonary infection. Typical pathogens include encapsulated bacteria such as strep pneumonia due to the lack of oxidizing IgG. Complement this function such as C3 deficiency also causes defective opsonization and infection due to encapsulated bacteria. Perirectal abscess, granulomas, and BCEPACIA infection or not expect it. Impaired airway clearance is a feature of cystic fibrosis, which can predisposed to BCEPACIA pneumonia, in addition infections and CF are limited to the Sino pulmonary tract. Impair neutrophil chemotaxis is seen in leukocyte adhesion deficiency. Patients are susceptible to recurrent rash and mucosal infections.

WK5 WED--------->cgd

Chronic granulomatous disease presents with recurrent cutaneous and pulmonary infections with catalase positive organisms such as staph aureus and Serratia. Abnormal oxidative burst such as dihydroergotamine testing is consistent with the diagnosis, and antimicrobial prophylaxis with TMPSMX and itraconazole is required IVIG/intravenous immunoglobulin replace immunoglobulins in patients with low levels such as excellent agammaglobulinemia, a finding that is not present in CGD moreover patients with low immunoglobulin levels typically have recurrent Sino pulmonary infection. Granulocytes stimulating factor is used to increase neutrophil counts in patients with low soluble neutrophil count such as chemotherapy induced neutropenia or cyclic neutropenia. I

WK5 SAT---------> Atrial fibrillation and hypertension

Chronic hypertension is the most common core mobility associated with a true fibrillation and is likely one of the strongest contributors to the predisposing a true remodeling. Aging is also a strong contributor.

WK4 SUN---------> delayed sleep wake disorder

Circadian rhythm sleep wake disorder characterized by Enterra great shift of the sleep wake cycle by more than two hours such as individuals go to bed very late in the evening and wake up very late in the morning. If the individual attempt to go to bed early he or she has sleep on said insomnia, if the individual wakes up to early here she has daytime sleepiness.

WK5 WED---------concussion

Concussion is caused by rapid rotational acceleration of the brain during head trauma. Diagnosis is based on clinical findings of neurologic disturbances such as headache, noise sensitivity, emotional changes without evidence of structural intracranial injury. Treatment is physical and cognitive rest followed by a gradual return Tension headache is a common cause of bilateral headache and contrast to a concussion tension headaches are not typically associated with light or noise sensitivity Internet for broadband visual task.

WK4 THURS--------->approah to diagnosis septic shock

cardiac index means cardiac output

WK4 TUE--------->defib v synchronized cardioversion

DEFINITIONS Defibrillation (DF) and cardioversion are methods of delivering electrical energy to the heart through the chest wall in an attempt to restore the heart's normal rhythm. There is an important distinction between defibrillation and cardioversion: Defibrillation — Defibrillation is the asynchronous delivery of energy, such as the shock is delivered randomly during the cardiac cycle. Cardioversion — Cardioversion is the delivery of energy that is synchronized to the QRS complex. Defibrillation and cardioversion may be accomplished using a manual defibrillator, which requires users to recognize the dysrhythmia and preselect the energy to be delivered. Alternatively, automated external defibrillators (AEDs) may be used. AEDs are computerized machines that automatically diagnose ventricular fibrillation (VF) and use voice prompts to instruct rescuers to defibrillate, if appropriate. In addition, based on preset values for heart rate and R-wave morphology, AEDs may advise defibrillation for ventricular tachycardia (VT).

WK4 SAT--------->dacryopcystitis

Dacryocystitis is an infection of the lacrimal SAC due to obstruction of the nasal lacrimal duct. It is characterized by pain, swelling, tenderness, and redness in the tear SAC area. Mucus or pus can be expressed

WK5 TUE---------> 68-year-old woman presents to the office due to worsening shortness of breath she has bothersome dyspnea on exertion and now developing with even light house work her oxygen saturation is 94% pulmonary function test shows that FEV1 is 65% of predicted in FEV1 to FVC ratio is 50% she has COPD what is the best next step in management?

Daily long acting muscarinic antagonist are the best initial therapy for stable COPD with a higher symptom burden such as dyspnea on light exertion and or disease risk such as history of exacerbation and hospitalizations. Inhaled corticosteroids such as budesonide are preferred for asthma reversible obstruction or as part of the triple therapy labor plus llama plus I CS for refractory COPD. Inhaled corticosteroids are not first line for COPD because llamas are more effective and half fewer Evers affects. Inhale corticosteroids increase pneumonia risk and COPD due to local pulmonary immuno suppression. Look a train antagonist montelukast are in a juvenile therapy for asthma rather than COPD Oral cortical steroids are indicated for eggs for acute exacerbation of COPD increasing increased such as increased wheezing sputum in this mirror but not as initial therapy for stable disease.

WK5 SUN---------> Diabetic kidney disease

Diabetic kidney disease is a microangiopathic glomerular process suggested by persistent proteinuria more likely occur in long-standing diabetes mellitus, sub optimal glycemic in blood pressure control and other systemic micro vascular damage such as retinopathy is associated with increased risk for cardiovascular mortality.

WK5 SUN---------> Diabetic nephropathy

Diabetic nephropathy is characterized by glomerular hyper filtration basement membrane thickening and mesangial nodules. Persistent proteinuria in a patient with with long-standing diabetes and poorly controlled hypertension should raise suspicion for these diagnoses.

WK5 WED---------> Diverticulosis

Diverticulosis is associated with chronic constipation it is usually asymptomatic but potentially include hemorrhage and diverticulitis. The risk of complications is lower with increase intake of fruit and vegetable fiber it is higher with heavy meat consumption aspirin or NSAID drug use obesity and smoking

Dobutamine

Dobutamine is a potent inotropic agents with a strong affinity for beta-1 receptors and a weak affinity for beta to an alpha one receptors. Dobutamine stimulates increased myocardial contractility leading to improved ejection fraction, reduced left end diastolic volume, and symptomatic improvement of decompensated heart failure.

WK5 SAT---------> treatment for non-ST elevation myocardial infarction

Dual antiplatelet therapy which is aspirin and a P2 Y12 receptor blocker leads to a reduction in recurrent myocardial infarction and cardiovascular death compared to aspirin alone and patients with non-ST segment elevation am I. It also reduces the risk of stent thrombosis and is recommended in all patients for at least 12 months following drug eluting stent placement.

WK4 SUN--------->Cavernous sinus thrombosis

Due to infections originating in the maxillary space which can spread to the inferotemporal space and orbit and progressed to cavernous sinus thrombosis.

WK4 FRI--------->43 yo male presewnts w/ blistering rash...severe pruritis...no exposure to chemical

Dyshidrotic eczema acute Palmer planter eczema is characterized by a recurrent puretic vesicular rash that primarily affects the palms and soles and sides of the digits. The diagnosis is based on clinical features and treatment includes high and super high potency topical steroids.

pfevention if the alloimmunization

Ectopic pregnancies can cause Rh(D)alloimmunization in Rh(D)-negative women due to fetomaternal blood mixing and maternal antics antibody production. Therefore, Rh(D) negative women require ant-D immunoglobulin after an ectopic pregnancy

WK4 SUN---------> attorney 60 woman presents six week postpartum for her check up she says the baby is feeding well and she's breast-feeding but she's been feeling really sleepy and tired. What should the patient be screened for?

Edinburgh postnatal depression scale -Postpartum depression has a high incidence and is often under reported. Therefore, all women regardless of prior psychiatric history require screening for postpartum depression such as the Edingburg postnatal depression scale.

WK4 SUN---------> A 20-year-old man comes to the office due to recent shoulder dislocation he has had multiple joint dislocation since childhood he also notes easy bruising and poor wound healing what is the most likely diagnosis?

Ehlers Danlos syndrome Ehlers-Danlos syndrome is a collection of genetic connective tissue disorders. It is usually characterized by joint hypermobility/laxity, multiple joint dislocation, tissue fragility, poor wound healing, and cigarettes, paper like scarring. Osteogenesis imperfecta is a genetic disorder of type one collagen which leads to bone fractures with minimal trauma. Marfan syndrome is an autosomal dominant disorder caused by mutation to the fibrillation one gene which results in connective tissue defects. Although patients have frequent joint laxity they also have excessively long bone growth leading to tall height and aortic root disease such as dilation aortic regurgitation aortic dissection. Klinefelter syndrome is seen in males with an X chromosome it is usually the result of primary hypogonadism such a small testes severely low sperm count, infertility, lesson visualization.

WK5 MON---------> paralytic ileus

Electrolyte abnormalities such as hypokalemia and hypophosphatemia which cause muscle dysfunction can cause paralytic ileus. Ileus typically appears on x-ray as generalized uniform distention of the large and small bowel.

WK5 WED---------> Emphysematous cholecystitis

Emphysematous cholecystitis is a life-threatening form of acute cholecystitis that occurs more commonly in patients with immunosuppression such as diabetes or vascular disease. It arises due to infection of the gallbladder wall with gas forming bacteria and requires emergency cholecystectomy. Acute cholangitis usually from common bile duct obstruction is also characterized by high fever right upper quadrant pain jaundice and is more common much more common Charcot triad, significant elevations an alkaline phosphatase and conjugated bilirubin would be more likely an imaging would show bile duct dilation without gas in the biliary tree or gallbladder wall. Billerey enteric fistula are extremely rare and typically occur due to pressure alteration from chronic gallstone disease not a cute cholecystitis. Large bowel fistulous are characterized by bile acid diarrhea where is fistulous with small bowel are often asymptomatic but may cause gallstone ileus which presents with symptoms of intermittent power obstruction such as nausea diffuse abdominal pain over several days after gallstones move through the intestines. Peptic ulcer perforation would be associated with free air under the diaphragm.

WK5 MON---------> A three year old girl is brought to the emergency department due to difficulty breathing she had developed fever and cough five days ago your an output has decreased her blood pressure is 86/50 her pulse is 150 her respirations is 36 sub cost to retractions are noticed leukocyte count is 17,000 chest x-ray reveals a right lower lung opacity and pleural effusion fluid analysis shows pH 7.1 glucose 30 proteins 4.0 lactate dehydrogenase 1100 leukocytes 60,000 neutrophils 90% what is the most likely diagnosis?

Empyema An empyema occurs after bacteria pneumonia leads to bacterial colonization of the pleural fluid. Pleural fluid shows a pH of less than 7.2 low glucose level, neutrophil predominant leukocyte count greater than 50,000, and significant Lee elevated lactate dehydrogenase levels. A chylothorax occurs when there is diminished and there is disruption of the thoracic duct leading to accumulation of Kyle often secondary to trauma or malignancy in the pleural space. Fluid analysis of chylothorax typically show elevated triglyceride levels and lymphocyte. Heart failure causes a transudate of a fusion in addition with a fusion due to heart failure pleural fluid analysis show low protein levels less than 3 g/dL Malignant effusion is an exudative fluid containing malignant cells in the pleural space. Fluid analysis can have an elevated LDH and low glucose however there is typically a low leukocyte count less than 5000. Mycobacterium tuberculosis a fusions typically present with chronic symptoms. Although pleural effusion may show low glucose and pH, leukocytes are low with a lymphocytic predominates.

WK5 MON---------> A 24-year-old woman at 10 weeks gestation presents with nausea and severe vomiting blood pressure is 90/50 she reports worsening chest discomfort that radiates to her back and retrosternal crunching sound is heard with each heartbeat what is the most likely diagnosis?

Esophageal perforation Protracted vomiting can cause esophageal rupture such as Borges syndrome. Patients typically have severe chest/back pain and may have pneumomediastinum the carpet is or a precordial crunching sound on auscultation known as the Hamman sign esophageal perforation is a surgical emergency

WK5 WED---------> Kidney stones

Excess urinary excretion of uric acid and low urine pH can lead to super saturation of urine and the formation of uric acid stones. Risk factors include gout, obesity, diabetes, metabolic syndrome, chronic diarrhea due to acidification of the urine and increase systemic uric acid production. Our colonization of the urine with potassium citrate effectively dissolves the stones. Malabsorption of fats such as Crohn's disease leads to increase binding of calcium by fat in the intestinal lumen which causes less calcium to be available to form insoluble oxalate complexes, facilitating increased intestinal absorption of oxalate. Renal clearance of the excess exsultate leads to calcium oxalate stones. However decreased oxalate absorption would not increase the risk. Increased the release of calcium and phosphorus from bone in hyperparathyroidism can increase the risk of calcium oxalate or calcium phosphate stones. Although Reno tubular reabsorption of calcium is increased net urinary calcium excretion is increased due to excess filter calcium though. Hydrolysis of urea yields ammonia which alkalinize is the urine and facilitates the precipitation of magnesium ammonium phosphate struvite crystals, recurrent upper urinary track infections with urease producing organisms such as Proteus or klebsiella can cause struvite stones. Because of the large quantities of urea in urine these stones can grow rapidly and feel the renal calyces staghorn calculi.

WK4 SAT--------->best predictors for post op lung resection?

FEV1 and DLCO

WK5 SUN--------->falls in older patients

Fall risk is multifactorial, therefore fall prevention is best achieved using an individualized approach in which staff determine prevention strategies based on the patients individual risk factors rather than a summary risk score.

WK5 SUN--------->familial hypocalciuric hypercalcemia

Familial hypocalciuric hypercalcemia is caused by a mutation in the calcium sensing receptor. It is a benign disorder characterized by asymptomatic hypercalcemia, elevated or inappropriately normal parathyroid hormone levels, and low urinary calcium excretion. It can be differentiated from primary hyperparathyroidism, which has increased urinary calcium excretion, by urine calcium/creatinine clearance ratio P

WK5 SAT---------> Transfusion reactions timeline

Febrile non-hemolytic transfusion reaction the most common adverse transfusion reaction, is a benign occurrence calls by cytokine release from local sites within the donor blood products. During preparation of blood products, whole blood is separated into red blood cells and plasma. However, small amounts of local site debris remains in the red seal concentrate. During blood storage, the circle sides can release cytokines, which one transfused can cause transient fever, chills, MLS within 1 to 6 hours of transfusion. Management includes transfusion cessation to exclude others serious reactions and anti-pyretic administration. Lucco reduction of donor blood helps prevent febrile non-hemolytic transfusion reaction. Careful cross matching of blood prevents acute hemolytic transfusion reactions which is caused by mismatch donor and recipient blood. Symptoms include chills, flank pain, and gross hematuria within an hour of transfusion. Those with acute hemolytic transfusion reaction have a positive direct anti-globulin/coombs test, increased plasma free hemoglobin, and he hemoglobinuria Citrate which bunds ionized calcium is the main anticoaguling used for blood product storage and can cause hypocalcemia if a massive transfusion is required such as cardiac surgery. Calcium gluconate infusion is used to prevent or treat severe hypocalcemia. Similarly, warming the blood product is recommended during rapid, massive transfusion to prevent hypothermia and arrhythmia.

WK4 SUN---------> 44-year-old man with HIV presents to the hospital two weeks ago with fever and headache was diagnosed with cryptococcal meningitis he was treated with amphotericin B and Flew cytosine what medication should be prescribed following discharge?

Fluconazole Cryptococcal meningitis is an aids defining illness that primarily affects patients which city four count less than 100. Three phases of treatment are required to ensure eradication and prevent relapse: induction with amphotericin B and flu cytosine for approximately two weeks, consolidation with high-dose oral fluconazole for about eight weeks, and maintenance with low-dose oral fluconazole until CD4 count recovers to more than 100 for more than three months on anti-retroviral treatment. All those steroids are used to treat limit inflammation and some opportunistic infections such as PCP, tuberculosis meningitis, steroids do not improve outcome and patients with cryptococcal meningitis.

WK5 MON---------> Non-Hodgkin lymphoma types

Follicular lymphoma is a common form of non-Hodgkin's lymphoma. It generally presents in an older patient with months or years of waxing and waning if I did not pate. The symptoms and laboratory abnormalities are rare but mediastinal/healer and then father and apathy is sometimes is sometimes seen

WK4 SUN---------> A nine-year-old boy presents to the emergency room due to lethargic headache and nausea since this morning he has had decreased urine output in the past 48 hours to weeks ago he had a sore throat today his blood pressure is 172/102 with Peri orbital scrotal and pedal edema present serum creatinine is 1.9 what is the next best step in management?

Furosemide Acute poststreptococcal glomerulonephritis is a complication of group a strep infection such as pharyngitis, typically presents with hematuria, Adema, and hypertension. Management is mainly supportive. Complications of volume overload such as hypertension edema require treatment with a loop diuretics. Some antibody mediated nephritic syndrome's such as Goodpasture are responsive to plasma pheresis.

WK5 SUN--------->graves opthalmopathy

Graves opthalmopathy is characterized by ocular irritation, impaired extraocular motion, and proptosis. It is caused by T cell activation in stimulation of orbital fibroblast by thyrotropin receptor auto antibodies, leading to expansion of orbital tissues.

WK5 TUE---------> hazard ratio

Hazard ratio is interpreted as follows: Has it ratio less than one means at any time that has it rate of the event is lower in the intervention group then in the control group such as a smaller portion of patients in the intervention group are experiencing an event compared to the control group Has it ratio equals one is the no value which means at any time that hazard rate of the event is the same in both groups Hazard ratio greater than one means at any time the hazard rate of the event is greater in the intervention group than in the control group. A confidence interval that includes to know value is not statistically significant and a confidence interval that excludes Danal value is statistically significant.

WK5 SAT--------->hidradenitis suppurativa

Hidradenitis suppurativa Is a chronic relapsing condition characterized by inflammatory occlusion of folliculopilosebaceous units. There is a strong association between tobacco use and HS.

WK4 FRI--------->36 yo M from california presents w/ HA..dizziness..sob..cough...fever..cyanosis...bl lung crackles for past 3 day while on vacation at skii resort in colorad..leukocytes 13k w/ 75% neutrophila..pt started inmproving w/ oxygen therapy -likely cause of sxs?

High altitude commentary edema typically presents with shortness of breath, cough, hypoxemia, and bilateral lung crackles. Characteristics that favor high altitude pulmonary edema over multifocal pneumonia include recent ascent to high altitude, the absence of leukocytes meaning less than 15,000, rapid improvement with supplemental oxygen. The presence of band cell would suggest bacteria pna

WK5 MON--------->

History of childhood sexual abuse and or adult sexual trauma has been consistently associated with the development of lifetime somatic symptom disorder is. Other significant risk factors for somatic symptom disorder include female sex, low educational level, chronic illness as a child and or childhood exposure to family members with chronic illnesses.

WK4 FRI--------->menopause

Hormone replacement therapy is indicated in the treatment of vasomotor symptoms such as hot flashes and night sweats and women under 60 years old who have undergone menopause within the past ten years

WK4 SUN---------> hyper somnolence disorder

Hyper somnolence disorder is characterized by persistent daytime sleepiness but no sleep attacks and is diagnosed only when excessive sleepiness is not better explained by another sleep disorder.

WK4 SUN--------->

Hypomagnesemia Hypomagnesemia is an important cause of hypocalcemia, particularly in alcoholics. Hypomagnesemia causes decrease release of parathyroid hormone and PTH resistance. Hypo parathyroidism induced by low magnesium is not associated with elevated phosphorus levels as seen with other causes of hypothyroidism. About 50% of circulating calcium is protein-bound predominantly to albumin. Consequently, serum total calcium varies with albumin concentration. Interpretation of serum calcium levels requires correct thing for serum albumin level serum calcium concentration falls by 0.8 mg for every 1 mg decrease in albumin. This serum dissipation serum calcium is low even after correct thing for the mild hypoalbuminemia. resistance to PTH such as pseudo hypo parathyroidism may result in hypocalcemia with high PTH and high phosphate levels. This condition is very rare and is usually associated with characteristic facial features such as all bright hereditary osteodystrophy.

WK5 WED---------> common causes of myopathy

Hypothyroid myopathy is characterized by myalgias, proximal muscle weakness, and an elevated serum creatinine kinase level. Patients often have the additional features of hypothyroidism such as fatigue delay reflexes. Initial diagnostic testing includes TSH and free T4 levels. Polymyositis is characterized by symmetric proximal muscle weakness. Sedimentation rate and CK are elevated in approximately half of patient, and the diagnosis can be confirmed with muscle biopsy. However myalgias are typically absent or mild and deep tendon reflexes are normal. Electromyogram and patients with hypo thyroid myopathy can be normal or shown a specific changes it is more helpful in evaluating patients with polymyositis and can be considered if thyroid studies are normal. Myopathy with elevated CK concentration usually is not a manifestation of lisinopril toxicity. The most common drugs associated with myopathy or corticosteroids statins and colchicine.

Hyper thyroidism

Hypothyroidism and anxiety disorders have considerable symptoms overlap including sleep disturbances palpitations and sweating. Measurement of thyroid functioning is the first step in management of patient with new onset anxiety and physical symptoms to prevent miss diagnosis and provide appropriate treatment.

WK5 SAT---------> Effects of hypovolemia in the renin in angiotensin aldosterone system

Hypovolemia stimulates the renin angiotensin system, which helps improves intravascular volume, glomerular filtration rate, and blood pressure via vasoconstriction and kidney mediated sodium and water reabsorption.

WK4 SUN---------> chancroid

I sexually transmitted disease caused by Haemophilus ducreyi. Patient present with one or more painful genital ulcers and painful, suppurative inguinal lymphadenopathy. Commonly seen in south east Asia, sub-Saharan Africa, and Latin America.

WK4 SUN---------> 58-year-old woman presents to the office with abdominal pain cramping bloating nonspecific to foods headaches fatigue insomnia joint pain vitals are stable her lab reports return normal we suspect somatic disorder what is the most appropriate issue statement by the physician?

I understand your symptoms are distressing. The good news is your normal test results indicate that serious medical diseases have been ruled out. When medical work up fails to identify a medical ideology in a patient with suspected somatic symptom disorder, the results should be communicated in a positive tone assuring the patient that there is no evidence of serious medical illness. The distressing and real nature of symptoms should be acknowledged first, followed by discussion of the suspected disorder and contributing psychological factors.

WK5 TUE--------->types of hepatitis

ISONIAZID—INH...not healthy in Neurons or Hepatocytes Alcoholic hepatitis lab show AST to ALT ratio greater than two an alcoholic hepatitis Auto immune hepatitis should be considered once all possible causes of drug induced or viral hepatitis have been ruled out. Positive anti-nuclear antibody and or positive smooth muscle antibodies suggest a diagnosis of classic type one autoimmune hepatitis. Tuberculosis hepatitis may develop in cases of military tuberculosis but will be characterized by granuloma on liver biopsy. isoniazid causes idiosyncratic liver injury with histological features similar to those scene with viral hepatitis.

WK5 SUN---------> pseudo thrombocytopenia versus idiopathic Thrombocytopenia purpura

Idiopathic thrombocytopenia Purpera which is approximately 10 fold more common in pregnant women than in the general population, usually presents with myATTThin but cytopenia. Patient are often asymptomatic and have no history of bleeding disorder. However, peripheral blood smear would show a paucity of platelets, not platelet clumpy. Pseudo thrombocytopenia is a laboratory era characterized by platelet aggregation in vitro it is generally confirmed when peripheral smear reveals large clumps of platelets. Patient do not require additional evaluation or follow up and are not at risk for bleeding.

WK5 SAT---------> Minor head trauma in the pediatric emergency department

Imaging should be avoided in children with minor head trauma and no high risk features for intracranial injury such as altered mental status loss of conscious severe mechanism of injury vomiting or severe headache signs of basal a fracture.

WK5 WED---------> Seizures

In emergency situations computer tomography scan of the brain without contrast is the initial imaging study of choice in patients with unprovoked first seizure to exclude acute neurologic problems such as intracranial or subarachnoid bleed that might require urgent intervention. MRI is more sensitive than CT in identifying most structural causes of epilepsy and is the Nuro imaging modality of choice in elective situation.

WK4 FRI---------post patrum period

In the immediate postpartum period, physiologic changes include uterine contraction, lochia, breast milk excretion and milk let down, and chills and shivering causing hyperthermia or low grade fever. These changes are hormone mediated such as increased oxytocin/prolactin levels, decreased estrogen/ progesterone levels. Patients with these normal findings are managed with routine postpartum care.

WK4 SUN---------> 45-year-old man presents with exertional dyspnea and fatigue he has an stage renal disease due to glomerulonephritis his blood pressure is 160/60 oxygen saturation is 96% room air appointment maximum impulse is displays to the left with the systolic murmur heard over the cardiac Apex Scarratt rails are heard at the lung bases and he has peripheral edema what is the likely cause of the patient symptoms?

Increased cardiac output High output heart failure usually results from increase cardiac output in response to a reduction in systemic vascular resistance such as atrial venous fistula. It involves hyperdynamic circulation evidenced by widened pulse pressure embrace carotid upstroke increase venous return leading to peripheral and pulmonary edema. Flail mitral leaflet leads to acute mitral valve regurgitation and subsequent heart failure. Because cardiac output is reduced, evidence of hyperdynamic circulation such as wide pulse pressure brace carotid upstroke and warm extremities. Hypertrophic cardiomyopathy is recognized by a systolic ejection murmur that increases in intensity with maneuvers day decrease left ventricular blood volume such as Valsalva.

WK5 SAT---------> Hepatitis B markers

Individuals with resolved hepatitis B infection or positive for anti-HBS and anti-HBC but -4 anti-HBS AG. Individuals who are immunized with the hepatitis B vaccine or positive for anti-HBS but negative for HPSAG and anti-HBC.

pyloric stenosis

Infantile hypertrophic parloric stenosis is most common in first born boys age 3 to 5 weeks who are fed formula. Non-bilous, projectile vomiting causes dehydration and weight-loss. Abdominal ultrasound confirmed the diagnosis.

WK4 SUN---------> A 58-year-old man with Burkett lymphoma says for the past two days after starting chemotherapy the patient reports nausea and generalized weakness. His lab results show sodium 140 potassium 6.8 blood urea nitrogen 24 glucose 110 ECG shows peaked T waves. In addition to calcium gluconate which of the following is the best initial therapy for this patient?

Insulin and glucose This patient with Berkenham Palma has hyperkalemia likely due to tumor lysis syndrome from chemotherapy. Common indications for emergency treatment of hyperkalemia include hyper Kaylee Mia related ECG changes such as peaked T waves or marketed elevation such as 6.5 or higher or a rapid rise and serum potassium level. Patients with these findings have hyperkalemic emergency and are at high risk for life-threatening cardiac events such a significant bradycardia sinus no dysfunction atrial ventricular block and ventricular arrhythmia. Hyperkalemic emergency is marketed by ECG changes, cardiac arrhythmia, and or sudden or dramatic rising serum potassium. Urgent treatment with calcium gluconate to stabilize the cardiac membrane and insulin plus glucose to shift serum potassium into cells is indicated. Definitive removal of potassium via the stool patiromer, kidneys such as furosemide, and or blood such as hemodialysis should then be initiated.

WK5 TUE--------->Levetiracetam

Is used for treatment of generalized seizures with generalized seizures you have loss of consciousness is usually accompanied by jerking movements tongue biting urinary incontinence and or post ictal confusion.

WK5 TUE---------> What is the biggest concern regarding per protocol analysis?

It will miss represent the fact of the intervention among patients of a medical practice Per protocol analysis include only those participants who strictly adhere to and complete the study protocol such as non-drop outs. This approach tends to provide an estimate of the true effect of an intervention for a perfect scenario, but he can overestimate the affect of the intervention in a practical clinical setting. Intention to treat analysis includes all participants as initially allocated after randomized, regardless of what happens during the study. Such as keeping the people who drop out in their retrospective groups for analysis intention to treat analysis estimates to affect of a signing rather than receiving an intervention intention to treat analysis could lead to a more conservative estimate of the affect of the intervention. It closely mirrors the expected number of drop outs in a real life situation so intention to treat analysis reflects the realistic it's expected effect of the intervention in a medical practice setting.

WK4 SUN---------> congenital long qt syndrome

JaVale and lanes Nielsen syndrome is a congenital long QT syndrome also causes inheritable sensory neural hearing loss due to the associated mutations affecting endolymph production. Congenital long QT syndrome typically present in childhood with profound bilateral deafness and episodes of arrhythmia induced syncope during periods of stress.

WK5 SUN--------->management if preterm labor

Labor is painful, regular contractions that call cervical change. False labor is mild, irregular contractions that cause no cervical change and ultimately resolve without intervention. Patients and false labor can be discharged home with labor precautions.

WK5 WED---------> Paraneoplastic syndromes

Lambert Eaton myasthenic syndrome is frequently associated with an underlying malignancy such a small cell lung cancer. It is caused by auto antibodies directed against voltage gated calcium channels in the presynaptic motor nerve terminal leading to symmetric proximal muscle weakness with depressed deep tendon reflexes. Myasthenia gravis is caused by auto antibodies against postsynaptic acetylcholine receptors and then neuromuscular junction. In contrast to Lambur Eden myasthenia syndrome patients with my myasthenia gravis typically present with fatigable ocular such as ptosis diplopia and bulbar such as dysarthria muscle weakness. Myasthenia gravis is classically associated with thymoma Immune mediated muscle inflammation suggest polymyositis. This condition can present with proximal muscle weakness. Multicentric central nervous system inflammation and demyelination are indicative of multiple sclerosis, which typically presents between in women between 815 and 50 with a neurologic deficit disseminated in space and time such as optic neuritis, internuclear ophthalmoplegia. Upper and lower motor neuron degeneration is suggestive of ALS. It generally presents for bulbar symptoms followed by limb weakness associated with upper hyperreflexia spasticity and lower muscle atrophy fasciculation motor neuron signs. Acute inflammatory demyelinating polyneuropathy such as Guillain-Barré is characterized by acute ascending muscle weakness with a reflexa most patients have a preceding viral illness.

WK4 SUN---------> A 68-year-old man presents to the emergency room due to sudden onset of right sided weakness there is weakness in the right lower face and right upper and lower extremities where is the location of the lesion?

Left internal capsule A stroke causing hemiparesis can usually be further located based on the associated symptoms. A pure motor stroke with no cortical signs or cranial nerve deficits is likely due to a lesion in the contralateral internal capsule. A stroke in the left frontal parietal cortex Khalid to right hemiparesis but a cortical stroke large enough result in face arm leg weakness would also likely result in since we disturbances in cortical sign such as visual disturbances aphasia and or neglect A lesion in the ponds would likely be accompanied by Ponting ipsilateral such as crossed signs cranial nerve palsy's in cranial nerves six through eight A stroke and impacting the right frontal parietal cortex with lead to left hemiparesis and cortical sign such as neglect, visual changes, sensory changes, aphasia and the dominant hemisphere A stroke impacting the right internal capsule could lead to pure motor stroke as seen in this patient the hemi paresis will be contra lateral to the lesion resulting in left him a paresis rather than right.

WK4 SUN---------> A 40-year-old woman presents with pain in her right hip for the past four weeks she has a past medical history of systemic lupus erythematosus which of the following is the best next step in management?

MRI of the hip Osteonecrosis is a common complication of SLE, and the risk is increased with chronic glucocorticoid use. He causes pain in the groin, thigh, or bud talk that is worsened by activity. Early examination findings and x-rays may be normal, and an MRI is more sensitive test.

WK4 MON--------->22yo F w pmh of T1dm..if evaluated in labor and delivery dt drowsiness...underwent 2-csection an hour ago dt preeclampsia...bp=148/98...pulse 70..RR=10..urine dark..blood glucose is 24..patellar reflex is absent BL -like dx?

Magnesium toxicity -w eo of hyporeflexia/areflxia..respiratory depression/drowsiness...need to tx w/ IV calcium gluconate -pt w/ mag toxicity typically have absent patellar reflexes and respiratory depress..toxicity presents w/ respiratory paralysis and cardiac arrest. Tx is cessation of mag sulfate and administration of calcium gluconate -acute blood loss/postpartum hemorrhage presents w/ hypoxia..dec urine output dt hypovolemia..tachycardia...tachypnea and hypotension -amniootic fluid embolism—an OB emergency that cause sudden cardiac collapse and possibly DIC...abnormal lung exa such as BL crackles..tahcypnea -pts w/ DKA will show neurologic deficit s when blood glucose is over 350 -pts w/ preeclampsia...are at inc risk foe eclamptic seizure...and an eclamptic postictal state can present w/ drowsiness...but usually have hyper-reflexia such as clonus

WK5 SUN--------->when imaging is necessary in dehiscence

Mediastinitis it is a complication of cardiovascular surgery characterized by infection of the deep tissues, classically presents for systemic symptoms such as fever tachycardia chest pain chest wall edema/carpenters and purulent discharge but it can also present a typically. Therefore any patient with copious drainage from the sterno launch undergo chest imaging. Radiographic findings include fluid collection or pneumomediastinum.

WK5 TUE---------> 64-year-old woman with chronic kidney disease due to hypertension and diabetes started taking EPO stimulating medication. Her creatinine has increased and her blood urea nitrogen has also increase what is the cause of the kidney damage?

Medication adverse effect Hypertension is a common adverse effect of erythropoiesis stimulating agents. Patients who receive large doses or experience a rapid rise and hemoglobin concentration are at greatest risk. Cerebral venous sinus thrombosis most commonly affects young women were hypercoagulability risk factors such as use of oral contraceptives and typically cause a headache with or without nausea and or vomiting. Papilledema, due to increased intracranial pressure. Although hypertensive emergency is due to pheochromocytoma it may present with headache and retinal hemorrhages signs and symptoms of hyper adrenergic state such as sinus tachycardia palpitations trimmer and diaphoresis. Tertiary hyper parathyroidism represents progression of the secondary hyperparathyroidism typically presents with chronic kidney disease of the parathyroid hormone secretion is no longer inhibited by the serum calcium level. Hypercalcemia is present and may cause mild hypertension due to vasoconstriction.

WK5 WED---------> meningococcal meningitis

Meningococcal meningitis is a potentially fatal infection that presents with rapidly progressive symptoms such as fever headache nuchal rigidity petechiae over 12 to 24 hours. If clinical suspicion is high, empiric vancomycin and ceftriaxone should be administered as soon as possible; lumbar puncture should not delay treatment.

WK5 MON---------> A 19-year-old woman comes to the office due to severe acne. She has no journalistic acne on the face arms and back and forehead. What will establish her diagnosis?

Menstrual history Severe nodular cystic acne may be the initial sign of hyper androgen -ism in patients with polycystic ovarian syndrome. The diagnosis is based on the presence of two of the three manifestations clinical or laboratory evidence of hyper antagonism, menstrual irregularities, and polycystic ovaries on ultrasound.

WK5 SAT---------> Orthopedic aspect of child abuse

Metaphyseal corner fractures occur when an extremity is port or twisted. These fractures are a red flag for child abuse and should prompt a skeletal survey to evaluate for additional occult fractures.

WK4 TUE--------->56 yo F presents chronic diarrhea..5 to 6 /day...for past several months-..pmh of OA and htn...meds include naproxen and HCTZ..colonoscopy bx reveals mononuclear cell infiltration of the lamia propria and subepithelai collage band -likely dx?

Microscopic colitis Microscopic colitis disproportionately affects women over 60 years old and is characterized by chronic watery diarrhea that may occur during periods of fasting and or at night. Colonoscopy reveals grossly normal appearing colonic mucosa however biopsy demonstrates a mono nuclear infiltrate within the lamina propria and a sub epithelial collagen band may be present. Risk factors include concurrent autoimmune conditions, cigarette smoking, and certain medication such as NSAIDs

WK4 SUN---------> minimal change dz

Minimal change disease is the most common cause of nephrotic syndrome in young children and presents with Adema secondary to protein urea and hypoalbuminemia. Diagnosis is classic and management is empiric corticosteroid therapy

WK4 SUN---------> mitochondria disorder

Mitochondrial disorders are maternally inherited and can present with diverse symptoms that may include hearing loss or imbalance. They typically involve multiple organ systems and myopathy is often present.

WK5 SAT---------> Reno abnormalities and sickle cell disease

Most patients with sickle cell lead normal healthy lives. Painless hematuria is the most common complication.

WK5 WED---------> Muscle pathologies

Myasthenia gravis is characterized by fluctuating, fatigable muscle weakness that worsens with repetitive motions and improves with the rest. Common symptoms include weakness of the proximal such as difficulty rising from chair, ocular such as diplopia or ptosis and bulbar such as dysarthria, dysphasia muscles. Pregnant and postpartum patients are at increased risk for developing the disease. Multiple sclerosis is common in younger women and can cause neurologic symptoms such as weakness and blurred vision in discrete, noncontiguous domains however symptoms typically relapse and remit slowly over days to weeks; fatigable weakness that fluctuates through the day is more consistent with myasthenia gravis. Diabetic immuno trophy typically causes focal pain, weakness, and muscle wasting in the proximal legs; it is associated with weight loss. Myotonic dystrophy typically causes persistent progressive weakness and wasting of the facial and distal musculature such as intrinsic hand. Myotonia such as delayed muscle relaxation after contraction is typical and fluctuating proximal muscle weakness is unexpected. Myopathies such as polymyositis and hyper thyroid myopathy typically present with persistent symmetric proximal muscle weakness that may be associated with mild muscle pain and tenderness. Postpartum thyroiditis is typically associated with the signs of hyper thyroid such as heat intolerance, weight loss, tremor.

WK5 SUN--------->mycoplasma pneumoniae

Mycoplasma pneumonia causes a typical pneumonia with indolent symptoms of headache malaise low-grade fever incessant cough and non-exudative pharyngitis. Chest x-ray often reveals interstitial infiltrates with or without a small Sarris pleural fluid. Empirical oral antibiotic such as azithromycin usually resolve the infection completely.

WK4 SUN---------> 23 rolled men present with irregular sleep for the past three months he has been feeling more fatigue since his girlfriend broke up with him and describes weird things happens as he fall asleep like hearing whispering voices and seeing colorful animals -dxdiagnosis?

Narcolepsy The patient's excessive daytime sleepiness relapse into sleep such as unintentionally falling asleep in class, hallucinations while falling asleep, and cataplexy such as transient loss of muscle tone and response to intense emotion are suggestive of narcolepsy. Intrusions of REM sleep phenomenon during sleep wake transitions may include hypnagogic on falling asleep and hypnopompic or awakening hallucinations as well as sleep paralysis such as inability to move immediately after awakening. Narcolepsy is characterized by excessive daytime sleepiness, cataplexy, and rim sleep related phenomena such as hypnagogic/hypnopompic hallucinations and sleep paralysis.

WK5 SUN--------->ARDS

Negative fluid balance decreases the formation of pulmonary edema and accelerates recovery from acute respiratory distress syndrome.

WK5 WED--------->airborne v droplet v contact

Neisseria meningitis reproduces primarily in the nasal pharynx, and infectious particles are regularly aerosolized through coughing and respiratory procedures. Droplet precautions should be ordered for all potentially infected patient until they have been treated with antibiotics for 24 hours. Infections that spread through direct physical contact with infected materials or on for might such a skin and enteric infections married the use of contact precautions with providers wearing both gowns and gloves and patients rooms. Mini respiratory viruses transmitted via both direct contact and respiratory droplets requiring both contact and droplet precautions Airborne Dash when aerosolized particles less than 5 µm they stay suspended in the air for prolonged periods. Such cases require airborne precautions such as negative pressure rooms, respiratory mask with minimum 95% filtering capacities such as N95 mask to present inhalation. Neutropenic precautions provide protection for patients at risk for infection due to profiled immuno suppression. The goal is to prevent exposure of the patient rather than the healthcare worker therefore no specific protective gear is required unless contact of bodily fluids is anticipated., Neutropenic precautions include strict hand hygiene, private rooms with high-efficiency air filtration, no live plants or animals, and avoidance of intrusive probes such as rectal thermometers or enemas. Standard precaution Dash a previously universal precaution should be followed in the care of all patients. They include hand hygiene before and after contact, use of safety gear when there is possible contact bodily fluids, respiratory hygiene cough adequate, and safe injection practices. Standard precautions are included with all other precautions.

WK4 SUN---------> A 20 year old woman presents to the office with bilateral sensory neural hearing loss her mother also had hearing loss in her 20s the patient has difficulty standing on 1 foot with her eyes closed what is the most likely cause of the patient symptoms?

Neurofibromatosis This young woman with bilateral sensory neural hearing loss associated with imbalance likely has bilateral vestibular schwannoma's. This is conjunction with her family history of hearing loss is characteristic of an F2 vestibular schwannoma present with hearing loss and imbalance. Bilateral, hereditary schwannomas are most often associated with NF2.

preeclampsia

New onset severe range hypertension such as a systolic of 160 or greater or diastolic of 110 or greater during pregnancy can occur in patients with preeclampsia with severe features. Severe hypertension increases maternal risk of stroke, pulmonary edema, and myocardial ischemia, fetal risk includes abruptio placenta and intrauterine demise. Acute antihypertensive therapy with hydralazine, labetalol, or nifedipine is required. Labetalol is contra indicated in bradycardia such as a heart rate under 60 because it will further decrease to heart rate resulting in dizziness or lightheadedness.

WK4 SUN---------> A 45-year-old man presents for his diabetes follow up. He was diagnosed four years ago. His vitals are good and his physical exam is good is hemoglobin A1c is 7% and creatinine is 0.9 and his urine albumin to creatinine ratio is 15. What is the most appropriate next step?

No additional medication Patient with type two diabetes should be screened annually for moderate increased albuminuria. Patient with hypertension and moderately increased albuminuria should be treated with ace inhibitors for secondary prevention of overt nephropathy. However ace inhibitors are not indicated for primary prevention in patients with diabetes who are normotensive and have normal albumin excretion.

WK4 SUN---------> 27-year-old woman presents with facial weakness in the left side of her face seem to be holding down juice leaked out the left side of her mouth weakness of the entire left side of the face including the forehead and incomplete I closure vital signs are normal and ear exam is normal which is the most appropriate next step and diagnosis?

No additional testing is needed Bell's palsy a peripheral neuropathy involving the facial nerve cranial nerve seven presents with acute progressive unilateral facial weakness. In patients with a classic presentation, no further diagnostic work up is needed, and many other causes of facial weakness can be eliminated through a thorough history and physical examination. GBS should be evaluate it with lumbar puncture which often shows albumin cytologic association. MRI of the head may exclude stroke in patients with facial palsy....risk factors include advanced age hypertension or findings such as sparing of the upper face onset over seconds or minutes are suggestive of stroke

WK4 SUN---------> A 58-year-old woman presents to the office requesting to be screened for ovarian cancer after learning that her friend and a distant relative was diagnosed with ovarian cancer what is the most appropriate approach?

No screening indicated Family history inc the risk for ovarian cancer. For low or average risk patient, ovarian cancer screening is not indicated because it is ineffective for early diagnosis or reduction in mortality. In women with a family history of suggestive of hereditary cancer syndrome, testing for BRCA 1/2 and lynch syndrome can be offered.

WK5 SUN--------->enuresis in children

Nocturnal enuresis secondary to obstructive sleep apnea should be considered in a child who has bedwetting in addition to inattention, behavioral concerns, hypertension, and or tonsillar hypertrophy. Evaluation is with nocturnal polysomnography.

WK4 MON--------->22yo F who's 8wks GA gets a thyroid fin tes....6months ago TSh was 2.0...T4 was 8...T3 was 130...today TSH dec to 0.3...T4 inc to 15...and t3 inc to 220.. -likely cause?

Normal physiologic changes - Thyroid hormone production increases during pregnancy to cope with metabolic demands. Estrogen causes an increase in thyroxine binding globulin, leading to increased total thyroid hormone levels. HCG directly stimulates TSH receptors causing increased production of thyroid mounds. Euthyroid sick syndrome is an alteration in bio chemical thyroid function test in the setting of severe non-thyroid illness. Common findings include a low total and free T3 With a normal T4 and TSH. Graves disease causes an increase in total and free thyroid hormone levels. Common findings include a low total and free T3 with a normal T4 and TSH.

WK4 SUN---------> A 34-year-old woman at 30 weeks gestation presents to the clinic for evaluation of an abdominal bulge. She performs the Valsalva maneuver and it and abdominal mass protrudes between the rectus abdominis muscles. Next best step in management?

Observation and reassurance Rectus abdominis diastasis is a weakening of the linea alba between the rectus abdominis muscles that can present as a nontender abdominal bulge in pregnant or postpartum patients. Management is conservative with observation and reassurance.

WK4 SUN---------> A 77-year-old man presents to the office for health maintenance the patient is doing well but reports to has a 30 pack your smoking history but quit 15 years ago what is the most appropriate next step in the management?

Obtain abdominal ultrasound The primary risk factors for abdominal aortic aneurysm include male gender, smoking history, age over 65. Do US PSTF recommend screening for this condition with a one time abdominal ultrasound in men between 65 and 75 who ever smoked.

Evaluation of occult gastrointestinal bleeding

Occult go bleeding Usually presents with unexplained iron deficiency anemia and or a positive fecal occult blood test. Initial work up includes colonoscopy and upper endoscopy, small bowel evaluation such as video capsule endoscopy, deep in Terrell's copy may be required if initial test are unrevealing. The presence of hemorrhoids should not preclude endoscopic evaluation. Pt still needed colonoscopy although he had hemmroids

WK4 SUN--------->Carotid artery rupture

Occurs when the vasa vasorum supplying the carotid is injured, it is most often a complication of head and neck cancer such as tumor invasion, tissue removal.

WK5 TUE---------> types of tremors

Of a functional psycho genic tremor often present abruptly with significant disability. They often decrease with distraction have changeable or inconsistent features and are not consistent with known tremor syndromes a functional tremor is typically low amplitude and relatively fast 10 to 12 hurt. It also typically worsens with movement and improves with rest. Orthostatic tremor typically affects the legs and trunk rather than the hand and occurs only one standing Although essential tremor is often seen more prominently in the hands and can be worsened by anxiety it most often present in older adults as a tremor of the hand that is suppressed at rest exacerbated by outstretched arms and more pronounced at the end of goal directed movement. A cerebellar tremor are classically low frequency less than 5 Hz and high amplitude, as seen in this patient they typically increase as an action approaches a target rather than ceasing with activity they are usually accompanied by other cerebellar signs such as ataxia dysmetria impaired rapidly alternating movements.

WK5 TUE---------> A seven year old boy presents to the ER due to seizures physical exam reveals inguinal folds and have multiple freckles on the chest and back darkly pigmented macules with smooth borders what are additional findings?

Optic pathway glioma neurofibromatosis type one is an autosomal dominant in neuro cutaneous disorder associated with café au lait macules and axillary inguinal fracking . Patients with NF1 have an increase risk of seizures, learning disabilities, and optic pathway gliomas. Acoustic neuroma's such as vestibular schwannoma's or tumors that are typically developed bilaterally and patient with an F2 patients with NF to do not have café au lait Mackeys or axillary inguinal's freckling Intracardiac rhabdomyoma are classically seen in patients with tuberous sclerosis and inherited neural cutaneous disorder that presents with benign tumors in multiple organs. Classic features include Ashley spots angiofibroma's and shagreen patches. Testicular enlargement in the setting of seizures raise his concern for fragile X syndrome. Fragile X is an excellent disorder characterized by intellectual disability, large ears, long and narrow face, macro or kite-ism after age 8. Sturge-Weber syndrome is characterized by a triad of Portwine stain them on the face, ocular disease typically visual deficits or glaucoma, and leptomeningeal capillary venous malformation. Most patients with Sturge-Weber have seizures due to these intracerebral vascular malformation's.

WK4 SUN---------> 14 year old girl presents for a wellness visit she's concerned because she has not had a menstrual period her height and weight are at the fifth and 55th percentile's her blood pressure is 140/90 ...an blood pressure is 90/40 in the lower extremitiesShe has multiple pigmented spots scattered on her face and chest. Which of the following complications is the patient most likely at risk of developing?

Osteoporotic fracture Short stature, amenorrhea and coarctation of aorta features of turner syndrome. Patients are at increased risk of osteoporotic fracture due to estrogen deficiency from ovarian dysgenesis

WK4 SUN---------> A 34-year-old woman comes to the office for evaluation three months of amenorrhea she began experiencing irregular periods a year ago and now they have stopped she has been treating the past for Hodgkin's lymphoma via chemotherapy for five years what is the most likely findings?

Ovarian failure may occur secondary to chemotherapy and presents for amenorrhea and signs of estrogen deficiency such as vaginal dryness. Ovarian failure is characterized by increased FSH and LH due to the lack of feedback inhibition from estrogen.

WK4 SUN---------> A medical Director implement a strategic plan to reduce iatrogenic transmission of respiratory viruses in his clinic by offering in-service training to the healthcare workers adding more hand sanitizer dispensers, updating ventilation system, giving patients mask in the waiting room, educating patients and promoting vaccination.The strategic plan best represents which of the following approaches to increase safety?

Overlapping barriers to block safety hazards The Swiss cheese model recommends overlapping multiple barriers to increase system safety. The Swiss cheese model to pics each defensive barrier as having vulnerabilities akin to holes in a slice of cheese, therefore stacking together multiple interventions layers of the cheese can block hazards such as errors caused by fatigue, infectious pathogens from passing through the system to cause harm, adverse patient outcome. Repeated cycles of process improvement - continuous quality improvement methods such as plan do study at cycle sick to improve a process by repeatedly testing, measuring, and refining a test of change. Incorporation of heart stops against incorrect action - force functions also known as heart stops use physical design to prohibit incorrect action. For example, oxygen and nitric oxide gas containers can be designed with different, incompatible connectors, which eliminates accidental connections to the wrong cylinder. Automating high risk action - computerized automation can reduce errors in high risk, manually performed processes such as automated calculating of drug infusion rates with an infusion pump.

WK5 TUE---------> A 65-year-old man comes to the clinic complaining of progressively worsening dyspnea over the last two months serum protein is 6.5 and lactate dehydrogenase is 320 patient has signs of congestive heart failure which of the following is most consistent finding with the plural of analysis of this patient?

PH of 7.45 Congestive heart failure commonly causes transfer date of infusions bilateral 61% unilateral right side 27% and unilateral left side 12%. Normal pleural fluid pH is 7.6. Transudate of fluid usually have a pleural fluid of 7.4 to 7.55.

WK5 SAT---------> Evaluation of suspected unstable abdominal aortic aneurysm

Pain is the most common manifestation of abdominal aortic aneurysm, it can vary according to aneurysms location. Proximal AAA tends to cause upper abdominal flank or back pain. Asymptomatic, hemodynamically stable patients, the diagnosis is best made by abdominal CT. Spinal MRI is indicated for patients with acute lower back pain and red flag features such as focal weakness, bowel or bladder incontinence, and fever....these suggest serious pathologies of the vertebral column such as core compression or epidural abscess.

WK5 WED--------->more GI illnesses

Pancreatic cystic lesions are usually asymptomatic. Jaundice is unexpected in the pancreatic tail lesion due to the distance from the biliary tree Gallstone obstruction of the cystic that can cause biliary colic such as intermittent right upper quadrant pain or cholecystitis such as right upper quadrant pain echo cytosis and fever Pyogenic liver abscesses are identified radio graphically as multiloculated hypoechoic hepatic lesion that present with jaundice and weight loss. However fever and leukocytosis are expected and an enlarged gallbladder would be atypical Encapsulated Peri pancreatic fluid collection such as pseudo cyst occur in patients with a history of pancreatitis and I usually asymptomatic but can cause epigastric pain in early satiety pseudocyst are almost never located close enough to the biliary tree to cause obstruction therefore jaundice in gallbladder enlargement unexpected Portal vein thrombosis is most often associated with cirrhosis acute portal vein thrombosis usually presents with abdominal pain where is chronic portal vein thrombosis most often presents with variceal bleeding. Weight loss jaundice and palpable gallbladder is most consistent with pancreatic cancer. Tumors in the head of the pancreas campers in with weight loss jaundice and then tender distended gallbladder which is curvier sign on examination imaging Kindom demonstrate dilation about the intrahepatic and extrahepatic bile ducts as well as the pancreatic duct such as a double bubble sign

WK5 MON---------> more paralytic ileus

Paralytic ileus is defined as temporarily impaired peristalsis hypomobility of the gastrointestinal tract in the absence of mechanical obstruction Ideology include intra-abdominal surgery inflammation of intra-abdominal organs abdominal trauma medication such as anti-cholinergics and opioids endocrine or metabolic abnormalities such as hypothyroidism electrolyte disturbances such as hypokalemia neuropathy such as diabetes or spinal injury vascular diseases such as Miss mesenteric ischemia and sepsis

WK5 WED--------->lung cancer

Patients with Hodgkins lymphoma who are treated with chemotherapy and radiation are at high risk for a secondary malignancy later in life generally graded in 10 years after therapy. Solid tumors in the lung especially smokers, breast, and gastrointestinal system such as colorectal esophageal gastric tumors are most common. Aspergillus typically and fix a pre-existing lung cavity caused by tuberculosis, sarcoidosis, bronchial sis, or neoplasm. Manifestations include hemoptysis, cough, dyspnea, and fever. Imaging usually shows a cavitary mass with air in the periphery, not a solid lung lesion. In addition this patient has no history of underlying lung disease. Radiation induced fibrosis is a consequence of previous long erratic radiation. Dyspnea, nonproductive cough, and chest pain typically develops for 24 months after therapy. Chest x-ray usually shows volume loss with coarse opacities, not a solid lung lesion.

WK5 TUE---------congenital QT PROLONGATION

Patients with congenital long QT syndrome or address for polymorphic ventricular tachycardia that leads to syncope or sudden cardiac death, especially during periods of rapid heart rate and high sympathetic activity. Beta blocker such as propranolol or not a law damping sympathetic activity and shorten the QT interval at high heart rate to reduce the risk of these complications.

WK5 SUN---------> Right ventricular response to massive in submassive pulmonary embolism

Patients with massive pulmonary embolism usually present with signs of low arterial pressure such as hypotension and syncope and acute dyspnea, pleuritic chest pain and tachycardia. The thrombus increases pulmonary vascular resistance in right ventricular pressure, causing right ventricular hypokinesis and dilation and hypotension.

WK5 SAT--------->STIs

Patients with neisseria gonorrhea infection or at risk for coinfection with other sexually transmitted pathogens such as chlamydia HIV syphilis. Patient should be screened for these infections, counseled on safe sexual practices, and given appropriate antibiotics.

WK5 TUE---------> Cardio acls

Patients with persistent tachyarrhythmia narrow or wide complex causing hemodynamic instability should be managed with immediate synchronized cardioversion Intravenous intravenous antiarrhythmic meds such as amiodarone or procainamide can be used in patients with stable current or refractory wide complex tachycardia. Intravenous beta blocker such as metoprolol esmolol or calcium channel blocker such as wrap a meal or deal Tiahs them can be occasionally used in stable patient who failed to respond to initial vagal maneuvers and or intravenous adenosine. And synchronized cardioversion Sanchís defibrillation which provides a high energy shark at a random point in the cardiac cycle is used during resuscitation efforts in patient with pulseless cardiac arrest who have a shockable rhythms such as ventricular fibrillation, pulseless ventricular tachycardia.

WK5 WED---------> Primary dysmenorrhea

Patients with primary dysmenorrhea have cyclic lower abdominal pain during menses and a normal pelvic exam. First line treatment is with an NSAID drug common presentation is low midline spasmodic pelvic pain that often radiates to the back or inner thighs. Cramps occur in the first 1 to 3 days of menstruation it may be associated with nausea diarrhea headache and flushing. No pathologic findings on pelvic exam it's a diagnosis of exclusion once secondary dysmenorrhea has been ruled out. NSAID drugs topical heat therapy, combined hormonal contraception, progesterone progesterone IUD are treatments. Pelvic congestion syndrome typically presents as a doll ill-defined pelvic ache that worsens with intercourse or during long periods of standing. Those with pelvic congestion syndrome have pain prior to menses that is relieved by menses.

WK4 FRI--------->PPD TB

Patients with the newly diagnosed HIV should be screened for latent TB infection with the tubercule and skin test or interferon gamma release assay. A TST with induration of five millimeters or greater is considered positive in this population. In the absence of active TB manifestations such as positive chest X ray fever night sweats cough, treatment of latent TB with nine months of daily isoniazid + pyridoxine or 12 weeks of weekly rifapentine a high dose isoniazid and pyridoxine is curative.

mechanical ventilation

Peak airway pressure is the sum of airway resistance and plateau pressure. The plateau pressure is the sum of the elastic pressure and positive and expiratory pressure peep and is calculated by performing the end inspiratory hold maneuver. Peep is calculated with that and expiratory hold maneuver.

WK4 THURS--------->68 yo M presents w/ sudden onset of sever epigastric pain 2hrs ago..accompained by nausea..vom...CXR shows air under diaphragm..pmh...dm..htn..HF w/ reduced ejection fraction -likely cause?

Perforated ulcer Sudden onset severe abdominal pain with peritonitis and subdiaphragmatic free air on upright chest x-ray is a classic presentation a perforated viscus such as perforated peptic ulcer. Biliary colic can cause epigastric pain but the pain is typically postprandial when the gallbladder contracts against a blockage such as a gallstone cystic duct in response to fatty meal. In addition, biliary colic is typically associated with a benign abdominal exam and does not cause fever or pneumoperitoneum Acute gallstone pancreatitis can cause sudden onset of severe gastric pain fever nausea vomiting. Abdominal x-ray is often normal but gallstones or paralytic ileus due to surrounding inflammation.

WK4 SUN---------> A 48-year-old woman present for Kally sent to me immediately after a needle is placed into the intraperitoneal space and CO2 gas and stuff and so fellation is performed, severe sinus bradycardia and transient atrioventricular block is noted. Which of the following is the most likely cause of the observed intraoperative finding in this patient?

Peritoneal stretching Laparoscopic intra-abdominal surgery such as cholecystectomy requires insufflation of CO2 into the abdomen to create space for surgical maneuvering invisibility. The increase intra-abdominal pressure stimulates stretch receptors on the peritoneum that response by triggering an increase in vagal tone. Consequently patients can develop severe bradycardia, atrioventricular block, and sometimes a systole. CO2 embolization is a rare complication of CO2 in self lesion usually resulting from inadvertent insertion of an sulfation needle directly into the artery, vein or organ such as the liver. Effective patients typically develop in Oregon infarction such as arterial embolism or hypotension and obstructive shock venous embolism. Severe bradycardia and intraventricular block are not expected. Vena cava compression can result from high intra-abdominal pressure however monitoring of intra-abdominal pressure make significant compression unlikely in addition cava compression would lead to decrease venous return with a reduction in cardiac output and a reflexive increase in heart rate. Systemic absorption of intra-abdominal CO2 can occur during laparoscopic procedures, leading to an increase in blood PaCO2. This causes peripheral vasodilation with a reflexive increase in heart rate. Agents use for general anesthesia such as propyl fall commonly have a cardiac suppressive affect, therefore in adequate anesthesia is expected to cause tachycardia rather than bradycardia.

WK5 WED--------->types of stress test

Pharmacologic vasodilator stress testing such as adenosine causes a marketed increase in blood flow in normal coronary arteries and a relatively small increase in blood flow and stenotic coronary arteries. The difference in blood flow allows for a diagnosis of obstructive coronary artery disease due to reduced uptake of radioactive isotope and ischemic myocardium.

WK4 SUN---------> A 38-year-old man presented bilateral hand pain with intermittent swelling over the past six months he has a past medical history of type two diabetes serum alanine in aspartame amino transferase is are elevated. Which of the following therapies would be most benefit to this patient?

Phlebotomy Hereditary hemochromatosis arthropathy often resembles osteoarthritis the difference in age at onset under 40 years old, predilection for the second and third metacarpal phalangeal joints and wrists, and presence of chondral calcinosis. Management is primarily symptomatic such as acetaminophen, NSAIDs, therapeutic phlebotomy is necessary to minimize other systemic complications of hereditary hemochromatosis such as liver disease.

WK4 SUN---------> A 34-year-old woman had a vaginal delivery six weeks ago that was complicated with severe postpartum bleeding she presents with complaints of little energy and lacking milk production what is the most likely cause of her current symptoms?

Pituitary ischemic necrosis Sheehan syndrome which is postpartum ischemic necrosis of the anterior pituitary is potentially life-threatening complication of massive postpartum hemorrhage. It typically presents with a lactation failure prolactin deficiency as well as hypotension and anorexia secondary adrenal insufficiency. Postpartum thyroiditis is characterized by autoimmune inflammation and destruction of the thyroid within a year of delivery. Causes transient hyper thyroid symptoms such as weight loss palpitations tremor followed by self-limited hypothyroid state. Primary adrenal insufficiency manifests with similar symptoms such as fatigue, hypotension, and weight loss. Patient with primary adrenal insufficiency have hyper pigmentation due to increased melanocyte stimulating hormone. Pituitary apoplexy generally refers to spontaneous hemorrhage into the pituitary gland and occurs most commonly in patients with pituitary adenoma. Typical symptoms include acute headache, visual field defects, and decreased visual acuity. Symptomatic deficiencies of fertility hormones are less common.

WK5 TUE---------> Hysterectomy with placenta in situ

Placenta Accredo occurs due to direct placenta villi attached to the uterine myometrium. Patients typically have difficulty with placental detachment following fetal delivery and continued placenta bleeding due to intense adhesions to the uterine wall. Treatment is with hysterectomy with the placenta left in situ to prevent life-threatening maternal hemorrhage.

WK4 SUN--------->

Platelet dysfunction In patients with chronic kidney disease, play latest dysfunction is most common cause of abnormal hemostasis such as easy bruising or non-palpable Purpera. PT and PTT are normal, platelet count is normal or mildly low. Vasculitis can cause Purpera these are typically palpable due to inflammatory vessel injury. Non-palpable Purpera is typically caused by platelet or coagulation disorders Play latest distraction such as immune thrombocytopenia often presents for bleeding involving the skin or mucosal membranes. Play the counter usually less than 100,000 and bleeding typically occurs only when the count drops below 20,000. Early bruising is typical a factor eight deficiency hemophilia a but PTT is prolong a usually develops in early childhood. Bleeding complications occur in a cute disseminated intravascular coagulation DIC, a consumptive coagulopathy. However most patients have an acute risk factors such as sepsis or trauma, and PT and PTT are prolonged. In chronic DIC, coagulation studies may be normal, but these patients typically have thrombosis in a history of malignancy.

WK4 SUN---------> A 48-year-old man with extensive smoking history is evaluated for cough and hemoptysis the dr. is concerned that the patient may have lung cancer the patient is leaving for vacation soon what is the most appropriate response by the physician regarding his lab results?

Please set up an appointment as soon as you return so we can discuss your results. Delivering bad news is optimally done and a face-to-face meeting that allow the doctor to best assess the patient's emotional reactions and provide empathy and necessary support.

PCOS treatment

Polycystic ovarian syndrome causes infertility due to an ovulation and may present with irregular menses, enlarged ovaries, and signs of insulin resistance. Ovulation can be induced with letrozole and aromatase inhibitor in patients with PCOS.

WK4 THURS--------->Post cholecystectomy syndrome-

Post cholecystectomy syndrome- Persistent abdominal pain or dyspepsia either post operatively early or months to years late after the cholecystectomy. Ideologies include biliary such as retain common bile duct or cystic duct stone or extra biliary such as pancreatitis peptic ulcer disease causes. Abdominal abdominal imaging such as ultrasound followed by direct visualization such as ERCP or MRcP can establish the diagnosis and guide therapy towards the causative factor.

WK4 SUN---------> A 22-year-old woman presents at 34 weeks gestation is brought to the ER by ambulance due to sudden loss of consciousness followed by generalized shaking your analysis shows 3+ protein in her blood pressure was 145/98 which of the following is most likely the cause of this patient's right arm findings?

Posterior shoulder dislocation Violent muscle contraction such as seizure electro concussion injury can cause posterior shoulder dislocation. On exam the arm is healed and aduction and internally rotated, with flattening of the anterior aspect of the shoulder. X-ray shows loss of normal relation between the humerus head and the glenoid an internal rotation of the humeral head. Most posterior this locations are managed with close reduction. Anterior dislocation is the most common form of shoulder dislocation and is usually caused by a direct blow or fall on an outstretched arm. In anterior dislocation, the patient holds the arm slightly abducted and externally rotated. Todd paralysis refers to transient unilateral weakness following a tonic clonic seizure that usually result spontaneously. Radio nerve compression from trauma can occasionally be seen and shoulder dislocation's for most commonly occurs in the forearm. It often results in weakness of wrist extension and decreased hand grip.

WK4 SAT--------->A 31 year old woman at 12 weeks gestation presents with symptoms of a migraine she also has a past medical history of migraines which of the following is the most appropriate preventative therapy for this patient?

Propranolol First Line preventive during pregnancy is with beta blockers such as Propranolol or metoprolol but you have the best fetal safety profile. Calcium channel blockers such as verapamil are also effective in safe and pregnancy. However, prolonged beta blocker use can potentially cause growth restriction likely due to decreased placental perfusion from low blood pressure ergotamine and triptans such as Sumatriptan can be used to abort an acute migraine but are not typically used for migraine prevention. In addition these medications are not used in pregnancy due to risk of fetal anomalies with ergotamines and growth restriction with triptans and preterm labor for both of the medications

WK4 SUN---------> A 63-year-old man comes to the ER due to eight hours of constant epigastric pain with diffuse and severe of the past two hours one episode of vomiting containing bile but no blood the patient is has a past medical history of a fib and chronic kidney disease his temperature is 100 he's being -36 patient is in moderate distress due to pain. The abdomen is diffusely tender to palpation with rigidity guarding a rebound tenderness. Hemoglobin 9.5 glucose is 200 creatinine is 1.8 INR ratio is 2.1 which is elevated and upright chest x-ray shows small amount of free air under the right diaphragm nasal suction nasal gastric suction and IV crystallize and antibiotics are started and an urgent surgeries plan which of the following additional interventions is most appropriate at the moment?

Prothrombin complex concentrate Patient on warfarin who require urgent surgery with the high risk of bleeding, or those who are experiencing significant hemorrhage, should receive prothrombin complex concentrate and IV vitamin K. If PCC is unavailable, fresh frozen plasma plasma may be given.

WK5 SAT--------->sadpucksee

Rapid compression of the ball during blown abdominal trauma can cause a perforated viscus. When viscous perforation occurs within the retroperitoneum such as a duodenal tear, classic symptoms and signs such as fever diffuse abdominal pain may be delayed. Retroperitoneal free air may be present on the abdominal imaging.

Blunt trauma of the aorta.

Rapid deceleration can cause blunt thoracic aortic injury. Why didn't mediastinum on the chest x-ray should increase suspicion for BTA eyes and left-sided pleural effusion, hemothorax may be present from hemorrhage preceding containment such as hematoma. Strict blood pressure control and emergent surgical intervention are necessary to prevent re-bleeding and death. Myocardial contusion can cause hypotension tachycardia what is classically accompanied by arrhythmia or new bundle branch block. Myocardiorupture usually results in immediate death. Cardiac Tampa may result if the if the rupture is contained by the Perry cardium but would cause muffle heart sounds and desended neck veins.

REM sleep behavior disorder

Rapid eye movements sleep behavior disorder involves dream enactments that occurred during rim sleep due to absence of muscle Antonia. If awaken, patients become fully alert and can recall their dreams. In older patients, these behaviors may be a sign of neurod generation such as Parkinson disease or dementia with Lewy bodies.

WK5 SUN---------> Cerebral herniation

Rapid hematoma such as epidural expansion after head injury can abruptly increase intracranial pressure, compress the temporal lobe, and cause Uncle herniation. The first sign is typically an ipsilateral fixed and dilated pupil due to oculomotor nerve cranial nerve three compression. Contralateral hemi paresis is often seen due to direct compression of the ipsilateral cerebral peduncle. Accessory nerve cranial nerve 11 dysfunction can occur from lesions in the medulla, such as occlusion of the posterior inferior cerebellar artery, or from damage during surgical procedures involving the anterior lateral neck. Injury can cause paralysis of the ipsilateral sternocleidomastoid and trapezius muscles with scapular winging. The facial nerve cranial nerve seven originates between the ponds and the medulla injuries at that level can cause a lesion of the lower motor neuron resulting in weakness involving the whole ipsilateral face. The glossopharyngeal nerve cranial nerve nine originates in the medulla and is most commonly injured by compression from a nearby tumor, such as jugular foramen syndrome in the posterior fossa tumor. Vestibular cochlear tumor cranial nerve eight originates between the pons in the medulla, com common causes of dysfunction include compression from a vestibular schwannoma or injury within the internal acoustic canal leading to loss of hearing and or balance.

WK5 SUN---------> relative risk

Relative risk is a measure of association between an exposure and an outcome. Confidence intervals that contain the no value such as relative risk equals one indicate that the association is not statistically significant. Adjust a relative risk minimize founding and provide better estimates of associations.

WK5 SAT--------->AD polycystic kidney dz v RCC

Reno cell carcinoma is a malignant tumor that arises from the renal tubule epithelium. Important risk factors include smoking acquire cystic kidney disease, nephrolithiasis, and chronic acetaminophen use. Typical clinical features include hematuria, flank pain, flank mass, anemia, weight loss, and possibly paraneoppastic manifestations such as hypertension hypercalcemia or polycythemia. The clinical manifestations of autosomal dominant polycystic kidney disease generally correlate with a progression of cystic structural degeneration of the kidneys and include hypertension, polyuria, flank pain, and renal dysfunction. Extra renal manifestations include liver cyst and cerebral aneurysms.

WK4 SUN---------> rett

Rett syndrome is characterized by a period of normal development followed by regression of speech, loss of purpose for hand use, stereo typical movement, and gait disturbance. Other features include early deceleration and head growth such as microcephaly, breathing abnormalities, autistic behaviors, and seizures.

WK4 SUN---------> ra

Rheumatoid arthritis is typically treated initially with NSAIDs or systemic glucocorticoid such as prednisone to provide a cute system and relief. Therefore disease modifying antirheumatic drugs are initiated for long-term management such as methotrexate is commonly recommended, with a tumor necrosis factor inhibitor such as eat Tanner said and other biologic agency used for patients with refractory symptoms. Although our a commonly causes join spacing narrowing in the MCP joints x-rays typically show periarticular osteopenia and Bonnie erosions.

WK4 SUN---------> An 18-year-old woman comes to the ER due to fever severe headache nausea and vomiting she lives in North Carolina her temperature is 100.9 she has a diffuse maculopapular rash with numerous petechiae seen on the trunk extremity's hands and feet lumbar puncture shows glucose 50 protein 70 leukocytes 28 zero red blood cells what is the likely cause?

Rickettsia Rickettsia Rocky Mountain spotted fever is a tick borne illness caused by rickettsia rickettsia. Classic symptoms include a few days of fever headache followed by a maculopapular or petechial rash that may involve the palms and soles. Without empiric antibiotic treatment such as doxycycline, noncardiogenic pulmonary edema and shock can occur. Acute HIV infection often presents with constitutional symptoms and maculopapular rash on the trunk the neck and face not on the palms and soles. Most patients have a diffuse lymphadenopathy a sore throat and often mucocutaneous ulcers shock would be a typical. herpes encephalitis is usually marked by acute constitutional symptoms for neurologic deficit and or seizures. The herpes rash is vesicular and typically occur around the mouth or genitals. Lumbar puncture usually reveals significant red blood cells in the cerebral/spinal fluid.

WK4 SUN--------->

Right side precordial ECG The patient's presentation is concerning for right ventricular myocardial infarction. The right ventricle is involved in up to half of inferior wall am eyes which are recognized by ischemic changes in ECG leads to three and AVF and commonly present with atypical epigastric pain rather than classical substernal chest pain. Nausea is also common but when am I primarily affects her right ventricle, shortness of breath is rarely present to the absence of pulmonary edema because inferior wall MIs usually result from obstruction right coronary artery, which usually for supplies blood to the Sino atrial an HO ventricular knows, Brady arrhythmias such a sinus bradycardia are frequently present Raven circular myocardial infarction commonly presents with epigastric pain and nausea and is commonly associated with hypertension in bradycardia. Or right side it precordial ECG should be obtained to confirm the diagnosis. Upper right abdominal x-rays can recognize air on under the diaphragm and is indicated to evaluate of ruptured peptic ulcer. CT pulmonary angiography is indicated to evaluate for acute pulmonary embolism, which can present with hypotension it and ischemic changes in the inferior ECG leads. It presents with absence of it presents with shortness of breath and tachycardia.

WK5 SUN--------->risk

Risk is the probability of developing a disease over a certain period of time. To calculate the risk, divide the number of exposed subjects with a disease by the total number of exposed subjects such as all subjects at risk.

WK5 WED--------->sarcoma botryoides

Sarcoma botryoides I highly malignant Imbriano rhabdomyosarcoma a peaks and childhood less than under four years old the pathology is a clear poly Puedo Mas the resembles a bunch of grapes protruding through the vagina spindle-shaped sales and it's Desmin positive symptoms include vaginal leaking leukoplakia vaginal alteration with contact bleeding is mount alter is discharge It's a form of rhabdomyosarcoma it typically present during infancy as a polypoid or grape like mass protruding from the vagina with associated vaginal discharge and bleeding.

WK5 SUN---------> Scleroderma Renal crisis

Scleroderma renal crisis is characterized by acute onset of hypertension and acute kidney injury in patients with systemic sclerosis. The mainstay of therapy is ace inhibitors typically captopril which reduce renin angiotensin aldosterone system hyperactivity, improve renal function and normalize blood pressure.

WK4 MON--------->26yo F presents for prenatal visit..10wks GA...last pregnancy was 2yrs ago baby weighed 10lbs...she had irregular cycles and unsure of last cycle...BMI is 30 -nbsim?

Screen for gestational DM -pt is high risk for gestation DM w eo macrocosmic baby in last pregnancy..high BMI and irregular cycles are signs of PCOS..which cause insulin resistance..putting her at high risk of dm EO: All pregnant patients require gestational diabetes screening at 24 to 28 weeks gestation. However, early screening such as add the initial prenatal visit is recommended in patients with obesity and additional risk factors for undiagnosed pre-gestational diabetes such as macrosomic infant and polycystic ovarian syndrome.

WK4 SUN---------> 48-year-old woman presents for a wellness exam after four weeks ago her blood pressure was 150/90 at a dental visit today's visit was also 152/88 ..BMI is 28..what other evaluation should the patient be spraying for?

Screening for diabetes Patient with hypertension should be screened for diabetes with either fasting blood glucose or hemoglobin A1c. Even in the absence of hypertension, screening for diabetes is advised for patient between 40 and 70 with a BMI over 25. -duplex Doppler ultrasonography can identify Reno artery stenosis, which typically occurs in older men would die fuse atherosclerosis and causes severe or resistant hypertension requiring three or more medication. Other characteristic findings include recurrent flash pulmonary edema and unexplained renal failure after starting an ACE inhibitor. Plasma renin activity is used to evaluate primary hyperaldosteronism. However patients typically have severe or resistant hypertension, hypokalemia is common.

WK5 SUN--------->pna

Secondary bacteria pneumonia is the most common complication of influenza this should be suspected in any patient who develops worsening fever or pulmonary symptoms after initial improvement. Strep pneumonia and staff aureus are the most common ideologic organisms. Staff aureus pneumonia tends to cause rapid onset severe necrotizing pneumonia with high risk of death.

Congenital infections

Sensory neural hearing loss is the most common sequela of congenital CMV. To reduce the risk of hearing loss, antiviral therapy should be given to the neonate with symptomatic CMV such as hepatomegaly, jaundice,. Ventricular calcifications.

WK4 SUN---------> two month old African-American baby is brought to the ER due to persistent crying and refusing to breast-feed temperature is 101.3 liver edge is 1.5 cm below the costal margin the baby cries with palpation of the upper right thigh and resistance and resist movement along that hip his blood count shows hemoglobin of 9.1 glucose size of 20,000 neutrophils 80% what is the most likely diagnosis?

Septic arthritis The presentation of septic arthritis an inference can be subtle it may include fever lack of movement of the involve joint, excessive passiveness fussiness such as during diaper changes or asymmetric swelling. Leukemia can present with bone pain, fever, and leukocytosis thrombocytopenia and or anemia would also be expected. Developmental dysplasia of the hip presents with hip instability and asymmetric Thai creases. Sickle cell pain crisis can present with severe bone pain due to vaso-occlusion bicycle red blood cells. These episodes typically begin as early as six months when the production of defective hemoglobin a increases. A two month old baby hemoglobin is primarily fetal hemoglobin, making pain crisis unlikely. Transient Sinnoh virus typically present and will appear in children with hip pain after a recent viral illness. Patients are usually school-age and leukocytosis is not seen.

WK4 SUN--------->A 27-year-old man presents to the office due to anxiety and poor sleep he worries that he will say something stupid or embarrassing at his weekly staff meeting at work he has difficulty falling asleep what is the most appropriate pharmacotherapy?

Sertraline Social anxiety disorder is characterized by anxiety and fear of scrutiny and social situations resulting in avoidance, distress, and social occupational dysfunction. The preferred pharmacological treatment is a selective serotonin reuptake inhibitor or serotonin norepinephrine reuptake inhibitor.

WK4 SUN---------> A 28-year-old man with type one diabetes is being treated for DKA. Which of the following is the best index to monitor the response to treatment of this patient's condition?

Serum anion gap The most accurate markers indicating resolution of DKA are the serum ANION gap and serum beta hydroxybutyric levels. The eight and I ON gap estimates the unmeasured eight and I ON concentration in the blood returns to normal with a disappearance of ketoacid anions.

WK5 MON---------> burn wound sepsis criteria

Severe burns are often complicated by one infections and sepsis. Risk is increased to large burns greater than 20% body surface area. Gram-positive organisms are common soon after injury; gram-negative organisms and fungi are more common after five days. A change in burn one appearance or the loss of skin graft is often the first side of burn one infection.

WK4 SAT--------->A 22 year old woman who is pregnant presents with worsening abdominal pain she has a past medical history of sickle cell disease her vitals are normal she has taken hydrocodone tablets today with minimal relief what is the most likely cause of the patients presentation?

Sickle cell pain crisis the patient at 12 weeks gestation with abdominal pain is likely having an acute pain episode such as vaso-occlusive pain crisis due to sickle cell disease. Acute pain episodes are more common in pregnancy, likely because of increased metabolic demands and a hypercoagulable state. although the incidents typically increases with gestational age comma acute pain episodes can occur in the first trimester comma especially if there are other precipitating factors such as stress nausea vomiting and dehydration

WK4 FRI--------->skene gland cyst

Skene duct cyst Pelvic organ prolapse → Differential diagnoses Paraurethral gland cyst A retention cyst that results from obstruction, accumulation of fluid, and cystic dilation of the ducts that drain the Skene (paraurethral) glands. Located on the anterior vaginal wall near the opening of the urethra.

WK4 SUN---------> 62-year-old man needs to have surgery for direct inguinal hernia is medical history includes hypothyroidism seasonal allergies and several episodes of sinusitis the patient also smoke cigarettes and drink alcohol occasionally which of the following would most likely increases risk of post op wound infection?

Smoking history Smoking is associated with an increased risk of surgical site infection and poor wound healing. Current smoking is associated with greater risk than past smoking, but patients who quit are still at increased risk. Smoking cessation is recommended prior to elective surgery, especially if cessation can be achieved at least 4 to 6 weeks before surgery.

WK4 SUN---------> 28-year-old woman presents to the office due to panic attacks she's promoted seven months ago and has experienced severe anxiety such as trembling shortness of breath when giving presentations or meeting new clients. She says I have been anxious around people since I was a teenager what is the most likely diagnosis?

Social anxiety disorder Social anxiety disorder is characterized by fear of social situations and anxiety about embarrassment. In the performance only subtype, anxiety occurs only in performance related situations. So she anxiety disorder should be differentiated from panic disorder which is an unexpected panic attack in specific phobia which is a stimulus not related to social anxiety.

WK5 WED---------> Patient safety techniques

Stimulation based training prepares teams to respond effectively and rapidly during high risk clinical emergencies such as shoulder dystocia. This hand on training can improve team performance, refine technical skills, and reduce complications such as neonatal brachial plexus injury. Checklist that categorize patient by shoulder dystocia risk based on patient characteristics such as BMI and diabetes are ineffective for predicting or preventing shoulder dystocia because more than 50% of patients with shoulder dystocia have no identifiable risk factors as in this patient. Root cause analysis is a retrospective process performed after an adverse event to symmetrically identify all causes leading to the event. It is most useful for generating recommendations to address preventable errors. In contrast most shoulder dystocia's are unpredictable and unpreventable and require the routine practice of technical skills. Team day briefings are interactive discussions focused on safety concerns and actions in a specific situation. They can improve team communication and coordination but their benefits is limited to the team involved.

WK4 SUN--------->

Streptococcus agalactiae group B strep Strep throat caucus agalactiae group B strep or gram-positive cocci in pairs in chains on culture. It can cause early onset neonatal sepsis or late onset disease in young infants. Let onset infection may commonly present at age 4 to 5 weeks of bacteremia, meningitis, and or focal infection. Staff aureus infection should be suspected in a patient with cellulitis but staff aureus appears as gram-positive cocci in clusters. Late onset neonatal listeria can present with fever poor feeding and bacteremia in addition listeria is a gram-positive rod. And Tara caucus he calls is a gram-positive carcass that organizes and two pairs and chain simulator group B strep however it is much less common cause of late onset neonatal infection. Clostridium perfringens and corny bacterium species are gram-positive rods that cause gangrene is wound infections and respiratory problems such as diphtheria.

WK4 SAT--------->managing diabetic kidney dz

Strict blood pressure control particularly using ace inhibitors or arbs can reduce progression of diabetic kidney disease. Tight glycemic control with the target of HB A1C less than 7% also can prevent the progression of diabetic kidney disease. Further lowering is associated with an increased risk of hypoglycemia impossibly cardiac event period

WK4 SUN---------> An 18-year-old woman presents to the ER due to sudden onset of headache blood pressure is 156/90 CT scan shows no abnormalities CSF fluid analysis shows opening pressure of 260 glucose of 60 proteins 48 leukocytes five red blood cells 3500 what is the likely diagnosis?

Subarachnoid hemorrhage Subarachnoid hemorrhage classically presents with sudden onset severe headache lumbar puncture is warranted if clinically suspicion remains high despite normal CT scan of the head characteristic findings include an elevated opening pressure xanthochromia which is red blood cells in to see yourself and an elevated red blood cell count that does not decline with successive samples. Bacterial meningitis such as neisseria and herpes Symplex meningitis can present with acute headache and vomiting in adolescence. HSV encephalitis can also cause erythrocytes and CSF as well as temporal lobe necrosis however fever and elevated white blood cells in the CSF would be expected Idiopathic intracranial hypertension is most common in obese women of childbearing age and is characterized by headache vision changes and elevated opening pressure.

WK4 SUN--------->non treponemal serologic testing

Such as rapid plasma reagent is used to evaluate for syphilis, primary syphilis typically present with a single painless ulcer such as a chancre and bilateral nontender lymphadenopathy.

WK5 WED---------> Diverticular disease

Surgery is indicated for diverticula losses complicated by recurrent bleeding, perforation, peritonitis, or abscess test that does not resolve with percutaneous drainage. Uncomplicated diverticulosis can otherwise be managed none operatively such as with medication. Frequent colonoscopy is indicated for patient at increased risk for colon cancer such as large or multiple adenomatous polyps, Lynch syndrome. In the absence of Adenomas patients with diverticulosis may return to a routine calling screening schedule.

WK5 SAT--------->sydenham's chorea

Sydenham chorea Is an auto immune complication of group a strep infection caused by molecular mimicry in which anti-streptococcal antibodies cross react with Toronto antigens in the basal ganglia. Neuropsychiatric manifestations include chorea Milkmaid grip, hypotonia, emotional lability such as anxiety irritability obsessive compulsive behaviors.

WK5 SUN--------->fetal growth restroction

Symmetric fetal growth restriction begins in the first trimester and it's due to fear of conditions such as aneuploidy, congenital anomalies, and intrauterine infection.. Asymmetric fetal growth restriction occurs in the second and third trimesters and is due to maternal conditions that cause placentu insufficiency such as hypertension.

WK5 WED--------->sle

Systemic lupus erythematosus is a multi system inflammatory disease that can present with central nervous system findings such as generalized tonic clonic seizure which can also manifest as vasculitis thrombotic thromboembolic disease such as anti-phospholipid syndrome and transverse myelitis. Rheumatoid author rightists can cause symmetric hand arthritis low-grade fever anemia thrombocytopenia

Types of headaches

Tension type headaches can occur due to stress and are usually bilateral, dull, and non-throbbing without associated symptoms such as nausea and vomiting. When in frequent, they can be treated with analgesics such as ibuprofen, but when more disabling, profit prophylactic medication such as amitriptyline should be considered.

WK5 TUE---------> How do you deal with patients who are taking the medication and correctly or overmedicated?

The best way to prevent this type of error is providing plain language teaching Preventable adverse events are a major patient safety concern, risk increases with vague prescription instructions such as directed. Plain language medication teaching improves patient health literacy and reduce adverse events risk

WK4 FRI--------->more BPP

The biophysical profile is performed to assess fetal oxygenation through ultrasound observation and the non stress test period the components of the ultrasound assessment include measurement of amniotic fluid and observation of fetal tone, breathing movement and gross body movement. Each component of the BPP is scored zero or two and summed to give a score of zero to 10. A normal BPP is eight out of 10 or 10 out of 10 suggests that the fetus is well oxygenated. This patient has oligohydramnios such as a single deep pocket is less than two centimeters or and amniotic fluid index of five or less and an abnormal BPP score of four out of 10. A score out of zero out of 10 to four or out of 10 indicates fetal hypoxia due to placental dysfunction such as placental insufficiency. Risk factors for placental insufficiency include advanced maternal age, tobacco use, hypertension, and diabetes. The patient requires prompt delivery due to high likelihood of fetal demise.

WK4 SUN--------->A clinical trial is studying the fact witness of a new treatment to lower serum triglyceride levels and then in adults compared to the standard treatment. The goal is to have a power of 80% to detect a difference in mean serum triglycerides at the end of the trial the researcher reported a power of 92% what is the most likely explanation for these findings?

The difference in main serum triglycerides was greater than expected Power describes a studies ability to detect and affect such as difference between groups when one truly exist. Power is influenced by four factors Increasing sample size will increase power Decreasing outcome variability will increase power Increasing effect size will increase power Increase significance level Will increased power. -larger sample size is produce distributions that are tighter around the means than smaller sample sizes so the difference between distributions is easier to detect. Therefore with all other factors fixed, a larger sample size gives a study a greater power also outcomes with smaller variability makes the distributions tied around their means and give the study a greater power basically anything that's tighter around the mean will give a greater power

WK5 MON---------> A 52-year-old woman comes to the office due to several years of shortness of breath with exertion with a patient I had left side and early diastolic heart sound is heard followed by a diastolic rumble. Which of the following are the most likely hemodynamic findings in this patient?

The expected hemodynamic alterations of mitral stenosis are elevated pulmonary artery pressures with normal laughing circular pressures. The pulmonary hypertension is primarily passive, resulting from transmission of elevated left atrial pressure backward to the pulmonary circulation. There are many there may also be reactive components involving endothelium mediated Palmer arterial or vasoconstriction and remodeling.

WK4 SUN--------->failure and modes process

The failure modes and effect analysis process is a perspective error analysis tool that is often proactively used to optimize a new protocol or process. It evaluates the potential likelihood of errors at each stage in the process.

WK5 SAT--------->thyroid approach

The initial evaluation of thyroid nodules include a serum TSH essay in a thyroid ultrasound. Patients with a suppressed TSH undergo thyroid scintigraphy. Small, hyper functioning nodules are rarely make a malignant and do not usually require fine needle aspiration.

WK5 WED---------> Ventilator associated pneumonia And other respiratory issues

The intubated patient has increased respiratory secretions, worsening oxygenation, fever, tachycardia, and new pulmonary infiltrates on chest x-ray raising a strong suspicion for a ventilator associated pneumonia, a type of hospital acquired pneumonia that occurs more than 48 hours after intubation. Most cases arise due to aspiration of oral pharyngeal or gastric secretions. Ventilator associated pneumonia is usually suspected when an intubated patient develops several of the following New pulmonary infiltrates, increased respiratory secretions, signs of worsening respiratory status such as worsening oxygenation, low tidal volumes, and increase respiratory pressure, signs of infection such as fever, leukocytosis, and tachycardia a sampling of the lower respiratory tract is required for confirmation moderate or heavy growth of one or more microorganism is generally diagnostic. Acute respiratory distress syndrome is often seen in patients with severe trauma and frequently causes increased oxygenation requirements and pulmonary infiltrates. Most cases occur within 72 hours of the inciting event. Atelectasis due to mucus plugging is often associated with worsened oxygenation. Fever and plural effusion is less likely seen. Fat embolism is, with serious trauma of the pelvic and lung bones such as femur. Although it frequently causes acute respiratory decompensation, it generally presents within 24 to 72 hours of injury. Pulmonary contusion can occur in patients with blunt trauma to the chest and often causes increased oxygenation requirements. It generally presents within 24 hours following the injury it is not associated with increased secretions. Major pulmonary thromboembolism can cause acute respiratory decompensation it is not associated with increased secretions or low bar infiltrates Although volume overload can increase oxygen requirement and cause pulmonary infiltrates, fever and increase secretions are not commonly seen.

WK5 SAT--------->eating disorders

The key difference in differentiating anorexia nervosa from bulimia nervosa in adolescence is weight that is significantly lower than expected for age and developmental course. In contrast patients with bulimia nervosa typically maintain a normal weight. Signs of self induced vomiting includes parotid gland hypertrophy pharyngeal erythema dental caries, scars or calluses on the hand, and electrolyte abnormalities the binge/purge subtype of anorexia nervosa can be differentiated from bulimia nervosa by significant low body weight. The electrolyte abnormalities includes decrease potassium decrease sodium decreased chloride decreased phosphate decreased magnesium elevated bicarbonate elevated serum amylase..hypoalbuminemua...CBC will show pancytopenia

WK4 SUN--------- lean process

The main process improvement is the process of continually evaluating the methods used to identify and eliminate factors that use time, energy, and resources without improving patient outcomes.

WK4 FRI--------->Lymphocytic hypophysitis

The most common type of autoimmune pituitary inflammation (primary hypophysitis). Often occurs in late pregnancy or the postpartum period. Characterized by early lymphocytic infiltration and pituitary enlargement followed by progressive pituitary destruction, atrophy, and fibrosis. Can affect the anterior pituitary gland (i.e., lymphocytic adenohypophysis, which causes adrenal insufficiency, hypothyroidism, hypogonadism), the posterior pituitary gland (i.e., lymphocytic infundibuloneurohypophysitis, which causes diabetes insipidus), or both (i.e., lymphocyric panhypophysitis).

WK5 TUE---------> Number need to treat

The number needed to treat is the number of patients who need to receive a treatment to prevent one additional negative event number need to treat is the inverse of absolute risk reduction the lower the number need to be treat more affective the treatment because fewer patients need to be treated to prevent one additional negative event.

WK5 WED---------> Odds ratio

The odds ratio is a measure of association in case control study to compare the odds of exposure in cases relative to controls. The odds ratio is not a direct measure of risk.

WK5 SUN---------> Medical therapy for BPH

The preferred initial treatment for uncomplicated BPH includes Alpha one blockers, which can't provide a rabbit relief of symptoms. Five alpha reductase inhibitors can be used as an alternative or in addition to alpha blockers but have a much slower onset of action.

WK5 WED---------> ischemia of the bowel

The results from inadequate colonic collateral arterial perfusion to the left and sigmoid colon after loss of the inferior mesenteric artery doing aortic graft placement. Patient's present with abdominal pain and bloody diarrhea. Fever and leukocytosis may also be present. This adverse effect can be minimized hi checking sigmoid colon perfusion following placement of the aortic graft. Bow ischemia and infarction are possible early complications of operation on the abdominal aorta such as AAA repair. Aorta enteric fistula is a rare and late complication where the duodenum erodes into the proximal part of the aortic graft. Iatrogenic bowel perforation may be suspected after an abdominal operation. Signs of peritoneal irritation are usually present.

WK4 THURS--------->65 yo M w/ pmh of smoking has a 6cm aortic aneurysm...was sent to a surgeon for a repair...what should be discussed in the informed consent?

The risk of not undergoing the procedure - Essential elements discussed during the informed consent process includes the patient's diagnosis, the risk and benefits of both the proposed treatment and treatment alternatives and the risks of refusing treatment. It is not required or even feasible to cover all of the possible potential complications of procedures but the physician must discuss common complications in those that are very serious. Although some patients may want to know the financial cost of undergoing treatment, this is not an essential element that must be discussed during the informed consent process. This information is tip usually not available to the physician and is usually discussed with health insurance.

WK5 MON---------> fast ultrasound

The spleen is one of the most commonly injured organs in Blount abdominal trauma. Patients with severe splenic laceration causing intra-abdominal hemorrhage may have shock and intraperitoneal free fluid I'm focused assessment with sonography for trauma.

WK4 FRI--------->features of intrcranial htn

The symptoms of intracranial hypertension include headache worse that's worse at night nausea vomiting mental status changes. Papilledema focal neurologic deficits may be seen on examination. Cushion reflex which consists of hypertension bradycardia respiratory depression is a worrisome findings suggestive of brainstem compression

WK4 FRI--------->76 yo F being evaluated foe recurrent fever and cough..was admitted 7 days ago do to delirium from UTI...on pE crackle and rhonchi heard in right lobe...poor dentiton..fever..sputm gram stain shows gram+ and neg..and anaerboes unerdlying cause?

This patient developed fever cough and pulmonary infiltration after more than 48 hours after being admitted to the hospital indicating hospital acquired pneumonia. The presence of poor dentition and apical right lower lobe infiltrate and polymicrobial sputum gram stain results suggest an underlying cause is bacterial aspiration pneumonia risk is increased in hospital patients due to the following you such as use of sedative and antipsychotic medications, use of gastric acid suppression medications, glottal or pharyngeal disruption.. bacterial aspiration pneumonia is a common cause of hospital acquired pneumonia particularly in the setting of sedative or antipsychotic medications, gastric suppression medications, intubation, and aesthesia, or nasogastric feeding. Patients typically develop fever productive cough and infiltrate in a in a dependent portion of the lung

WK5 SAT---------> Hypothalamic pituitary ovarian axis immaturity

This teenager has abnormal uterine bleeding which is defined as during adolescence as menstrual bleeding less than 21 days or more than 45 days apart. And adolescents who have recently undergone menarche, immaturity of the hypothalamus pituitary ovarian axis fails to produce appropriate quantities and ratios of gonadotropin releasing hormone, and therefore LH and FSH, to induce ovulation. As a result, during the first few years post menarche, the majority of menstrual cycles are anovulatory and present as painless, irregular, heavy bleeding.

WK5 TUE---------> 59-year-old man collapse on the way to the emergency room a bystander witnessed the collapse and find no pulse and immediately calls for help what is the most important factor for survival in this patient?

Time to rhythm analysis and defibrillation if indicated Despite advances in therapy the overall survival rate for patients with sudden cardiac arrest remain quite poor. The most important factors in improving patient survival or prompt effective resuscitation with adequate bystander cardiopulmonary resuscitation prompt rhythm analysis and the fibrillation in patients found to be in a shockable rhythm such as ventricular fibrillation.

WK5 SAT---------> Optimal timing and mode of delivery after c-section with previous classical incision or myomectomy

Tobacco and cocaine use increases the risk of abrupt placenta via vasoconstriction resulting in placental ischemia and infarction and premature placenta separation from the uterus. Although abruptio placenta can cause abdominal pain vaginal bleeding and fetal bradycardia it is not associated with an irregular abdominal mass in addition neither tobacco nor cocaine increases the risk of uterine rupture. Prior uterine surgery particularly per a classic cc cesarean delivery is a significant risk factor for uterine rupture which can present with sudden onset abdominal pain vaginal bleeding fetal discoloration and a regular abdominal mass such as protruding fetal parts

WK4 SUN---------> A 58-year-old man present to the office with a hard mass on the posterior left elbow it has been growing for the past three years imaging shows at 3 cm soft tissue mass and bone erosion with overhanging edges of cortical bone at the Alex Reno in process. Serum creatinine is 1.7 what explains this patient's findings

Tophaceous gout Joe can cause a cute Barca prepatellar Alex Reno on inflammation, chronic versus swelling, AutoFest deposition in the bursa. Tophus induces chronic inflammation in the surrounding soft tissue in bond which can result in erosions in overhanging edges of cortical bone on imaging

toxic shock syndrome

Toxic shock syndrome caused by staph aureus bacteremia and associate exotoxin release typically presents with fever hypotension, tachycardia, and other fields exotoxin release, typically presents with fever hypotension, tachycardia and other fuse read macular rash. Treatment includes fluid replacement and antibiotic therapy with clients clindamycin plus vancomycin.

WK5 SAT---------> Proximal sympathetic hyper activity

Traumatic brain injury can cause damage to cortical areas responsible for inhibiting lower sympathetic centers. This disrupted inhibition may result in proximal sympathetic hyperactivity, a syndrome characterized by rapid onset episodes of tachycardia, hypertension, and tachypnea often accompanied by fever and diaphoresis.

WK5 WED--------->pancreatic cancer....head v the tail

Tumors in the head of the pancreas can present with weight loss, jaundice, and a nontender distended gallbladder Corvias sign on examination. Imaging can demonstrate dilation of both intra-and extrahepatic bile ducts as well as the pancreatic duct such as double duct sign. Symptoms of pancreatic cancer occurring at the head of the pancreas include painless jaundice such as yellowish skin pruritus pale stools dark urine and a non-tender distended gallbladder at the right costal margin call of year sign. In contrast cancerous in the body or tail of the pancreas usually present with abdominal pain but without jaundice since they do not typically obstruct the biliary system.

WK5 SAT--------->types of urinary incontinence

Urge—muscular/too strong detrusor Stress—weak/floppy detrusor muscle Stress incontinence...has nothing to do with the detrusor...its about hypermobility of the urethra which could be do to weakend pelvic floor muscles such as levator ani...so a mid urethral sling is used to stabilize the hyper mobile urethral sling

WK4 SUN---------> A 76-year-old woman presents to the emergency department with confusion daytime somnolence and decreased oral intake the patient is easily distracted and disoriented to time and place next best step in management?

Urinalysis Delirium is common but under recognize. The confusion assessment method improves delirium identification using its fundamental features of a cute asset, fluctuation, and attention, disorganized thinking, and altered consciousness. Initial evaluation is directed at the most common causes, but further assessment should follow any abnormal findings or negative initial evaluation. A CT scan of the head is indicated in patients with altered mental status with a history of trauma or anticoagulation or physical exam findings of focal weakness and Papel edema suggesting some kind of intracranial pathology Lumbar puncture is typically used to diagnose meningitis or encephalitis with findings such as fever or meningismus EEG can identify occult seizures as the cause of altered mental status. Patient with A concerning history such as stroke, trauma, critical illness or inadequate response to delirium treatment require EEG to rule out nonconvulsive seizures Ammonia is neurotoxic and elevated levels can cause delirium. Hyperammonemia most commonly results from acute or chronic liver failure such as cirrhosis which would be evident on initial evaluation such as jaundice aminotransferase elevation.

WK5 WED---------> Uterine inversion

Uterine inversion in which the uterine fundus inverts and prolapse through the cervix or vaginal after delivery typically appears as firm rounded mass protruding through the cervix or vagina. Patients can have severe abdominal pain heavy vaginal bleeding and a nonpalpable uterine fundus on abdominal examination. Uterine inversion requires discontinuation of utero tonics such as oxytocin and a mediate manual replacement of the uterus to prevent extenuation or maternal death. Uterine relaxants and laparotomy may be required if initial attempts at manual reduction are unsuccessful. So during uterine inversion the first step is try to manual replacement of the uterus.

WK4 WED--------->36 yo F at 35wks GA presents w/ painless bleeding and ROM...speculum exam reveals RM..the cervical os is 1cm dilated w/ minimal vaginal bleeding.

Vasa previa - Vasa previa is a rare condition in which fetal vessels overlie the cervix, making them prone to tear with rupture of membranes or the onset of labor. Patients with vasa previa may have painless, minimal vaginal bleeding and rapid fetal deterioration or demise

WK5 SUN--------->vascular rings

Vascular rings are congenital malformations of the great vessels that encircle and compressed the trachea and or esophagus. Respiratory symptoms may include biphasic strider that improves with neck extension. Esophageal symptoms include dysphasia, vomiting, and difficulty feeding.

WK4 SUN---------> A 23-year-old woman presents with three days of fever and dysuria. She has appear Riddick erythematous vulva rash that is painful during urination the left inguinal lymph node are enlarged and tender. Urinalysis is leukocyte esterase positive nitrates -0 bacteria and white blood cells 15 per HPF. Likely diagnosis?

Viral culture of lesion for herpes simplex virus Genital herpes Symplex virus infection can present with painful, pruritic, vesicular or ulcerative lesions, dysuria, and inguinal lymphadenopathy. Classical diagnosis requires confirmation with laboratory testing via viral culture or PCR. H. DUCREYI is a sexually transmitted infection that causes chancroid, which can cause multiple painful ulcers and tender and tender inguinal lymphadenopathy less common in women. However, the ulcers have a gray/yellow exudate and a friable base, and then lymph nodes classically undergo superation such as pulse. Diagnosis is via bacterial culture, Gram stain typically show gram-negative rods.

WK4 SUN---------> 69-year-old woman is brought to the office by her husband due to forgetfulness. She has difficulty remembering lesson plans for work and getting lost while going for walks in the neighborhood her mini mental exam was 22 out of 30 what is the most appropriate next step in evaluating a patient?

Vitamin B 12 level The initial work up of suspecting dementia should include neuropsychological testing such as Montreal cognitive assessment, selected laboratory testing complete blood count complete metabolic panel TSH, vitamin B 12, and neural imaging such as MRI. Patient and specific race groups may warrant additional targeted testing.

WK4 FRI--------->vulvodynia

Vulvodynia formerly termed vestibulodynia is a cause of painful entry dyspareunia. Physical exam shows pain to superficial touch of the vaginal vestibule.

WK5 TUE---------> Advance care planning for adolescence

discussions regarding advance care planning for adolescence with chronic life limiting illnesses such as cystic fibrosis should take family centered approach and directly involve the adolescent patient. Discussion should include information regarding prognosis, care options, and the patients and families preference for end of life care.

WK4 SAT--------->psych patient that stares mute and motionless

dx is catatonia..tx w/ benzos such as lorazapam and electroconvulsive therapy

WK4 FRI--------->cryptochidism..

even after surgical correction theres' still an inc risk of malignancy compared to general population

Hepatic adenoma

Young Women on prolonged oral contraceptives are at greatest risk for hepatic adenoma. Although most lesions are benign and asymptomatic, life-threatening complication such as malignant transformation or rupture can occur. Rupture should be suspected in the setting of sudden onset, severe right upper quadrant pain in size of hemorrhagic shock.

WK4 FRI--------->58 yo M presents w/ inc abd girth..abd US shows splenomegaly..moderate ascites...shrunken liver w/ no masses...diagnostic paracentesis reveals straw colored fluid -nbsim?

furosemide and spirinolactone work up of ascites include ultrasonography in diagnostic paracentesis such as serum ascites albumin gradient, cell count and differential, total protein to determine the underlying etiology in rule out spontaneous bacterial peritonitis. Initial management of ascites from cirrhosis include spironolactone and furosemide sodium restriction and alcohol abstinence.

WK4 SAT--------->bacterial keratitis

is usually seen in contact lens wears and following corneal trauma. Corneal appears hazy with a central ulcer and adjacent stromal Abscess. hypopyon may be present

WK4 FRI--------->bartholin glandcyst

most are asymptoimatic and self rewsolve...the symptomatic cases can be treated w/ incision and drainage..followed by a word catheter placement

WK4 FRI--------->salmonella

nontyphoidal begins w/ in 3 days of ingestions w/ sxs of fever..abd pain..diarrhea..it can be bloody or not

WK4 FRI--------->BPP features

normal fetal HR is bt 110-160bpm

WK4 THURS--------->when to intubate w/ asthma?

paCO2 over 50mmHg or PaO2 less than 50mmHg or severe cases ..cyanosis...inability to maintain respiratory effort..altered mental status...

WK4 FRI--------->post partum fever

post part. fever is 38 degeees C or 100.4F..excluding the first 24 hours

WK4 MON--------->20 yo female presents w/ dysuria for past 2 days...urine frequency but no chills..nausea..vomiting...urine culture grows e.coli -nbsim?

postcoital abx - A significant risk factor for recurrent cystitis such as two or more episodes in six months or more than three episodes in a year and sexually active women. Postcoital antibiotics prophylaxis such as nitrofuritoin, TMPSMX can reduce the rate of recurrent.

STDs

primary syphilis is marketed by the formation of a single painless Shanker that begins as a Papill and becomes a non-exodative ulcer with Indorated borders. Mild to moderate painless bilateral and fi the Napathy is often present. false negative non treponemal serology such as a rabbit plasma reagent is common in primary syphilis. Patients with negative serology and strong clinical evidence such as Schenker of the primary syphilis are treated empirically with intramuscular benzathine penicillin G.

WK4 THURS--------->approach for elevated ALP

pts w/ severe PBC w/ advanced live dz sxs such as cirrhosis or severe liver dmg will require liver transplant but mild sxs can be managed w/ ursodeoxycholic acid

WK4 SAT--------->63 yo F presents for a wellness visit...PE shows vagina atrophy..w/ minimal rugations..and an anterior bulge on valsalva.. -nbsim?

reassurance an observation pelvic organ prolapse may present w/ pelvic pressure...or urinary dysfunction..such as retention..stress urinary incontinence..however..many pts w/ POP are asymptomatic and are managed w/ reassurance

WK4 SAT--------->round ligament pain

round ligament being stretched dt a gravid uterus..;presents as a sharp pain that radiates to the uterus

WK5 TUE---------> exudate light criteria

translate date of fluid is usually due to systemic factors such as increased hydrostatic pressure or hypoalbuminemia and has a plural pH fluid between 7.4 and 7.55. Exudative is usually due to inflammation with a pleural fluid pH of 7.3 to 7.45. Pleural fluid pH less than 7.3 with a normal arterial pH and low pleural glucose is usually due to increased acid production by pleural fluid cells and bacteria such as empyema or decrease hydrogen ion reflux from the pleural space such as pleuritis, tumor, pleural fibrosis. Glucose less than 60 or plural or plural fluid/serum glucose ratio less than 0.5 is usually seen in complicated. Pneumonic a fusion, malignancy, tuberculosis, or rheumatoid arthritis. Transill dates and all other exudative effusion's usually have pleural fluid glucose similar to blank glucose concentration. High pleural fluid amylase level is typically due to pancreatitis associated effusion's or esophageal rupture from saliva. Plural LD age of 210 would give a pleural serum/LDH ratio greater than 0.6. Pleural proteins of five would give a plural serum protein ratio greater than 0.5. Both of these are found in exudative infusions

WK4 SUN---------> normal thyroid values?

tsh 0.4-4.0 T4 5-12(thyroxine) Free t4 0.9-1.7

WK4 MON--------->fetal heart rate monitoring

variable decel---sharp drop and return to baseline early decles--dt head compression--HR will align w/ mom's contractions late decels--dt placental insuff-->gradual drop and gradual return to baseline

WK4 SAT--------->65 yo M presents w/ severe interscapular back pain and chest pain after undergoing a TEE..CXR reveals widened mediastinum.. nbsim dx?

water-soluble contrast esophagography -esophageal perforation is a life threatening complication of esophageal instrumentation. Clinical presentation may include severe chest back pain fever and a wider mediasteinem on chest X ray water soluble contrast esophagography can confirm the diagnosis Severe chest pain radiating to the back and wide and mediastinum on chest xray are concerning for aortic dissection. which CT angiogram of the chest and TEE can both diagnose. However, aortic dissection often has other associated findings such as new aortic regurgitation murmur, asymmetric blood pressure upperEGD is often used to evaluate for sources of upper GI bleeding


Ensembles d'études connexes

Ch 25: Assessment of the Respiratory System

View Set

NASM 7th ed, Chapter 19: Speed, Agility, and Quickness Training Concepts

View Set

Health Assessment Prep U: Chapter 16- Assessing the Eyes

View Set

Anatomy and Physiology Exam 1 (SuperExam)

View Set

Chapter 18 (Part 3)- Eating Disorders

View Set

Ch 20 Forming & Operating Partnership, Tax 332 Chapter 20, ACCT 4343-Tax of Business Ch.9

View Set

M13 Chapter 12 Retrofit and Future Trends AUTI 142

View Set